You are on page 1of 49

MATHEMATICS

TARGET: JEE (ADVANCED) 2014

MEDIUM : ENGLISH

CRITICAL
QUESTIONS
BANK
SUBJECT : MATHEMATICS MEDIUM : ENGLISH

Contents
1. Questions 1 - 20

2. Answer Key 21

3. Hints & Solutions 22 - 45


QUESTION FORMAT & MARKING CRITERIA
SUBJECT : MATHEMATICS
A. Questions Format

In the booklet check that it contains all the 180 questions and corresponding answer choices are legible. Read carefully the
Instructions printed at the beginning of each section.
1. Section 1 contains 53 multiple choice questions. Each question has Four choices (A), (B), (C) and (D) out of which only ONE
is correct.
2. Section 2 contains 37 multiple choice questions. Each question has four choices (A), (B), (C) and (D) out of which ONE or
MORE are correct.
3. Section 3 contains 2 multiple choice questions. Each question has four choices (A), (B), (C) and (D) out of which Only ONE is
correct.
4. Section 4 contains 10 paragraphs each describing theory, experiment, data etc. Twenty questions related to Ten
paragraphs with one or two or three questions on each paragraph. Each question of a paragraph has ONLY ONE correct
answer among the four choices (A), (B), (C) and (D).
5. Section 5 contains 4 question. Each question contains statements given in two columns which have to be matched.
Statements in Column I are labelled as A,B,C and D whereas statements in Column II are labelled as p,q,r,s and t. The answers
to these questions have to be appropriately bubbled as illustrated in the following example.
6. Section 6 contains 1 multiple choice questions. Each questions has matching lists. The codes for the lists have coices (A),
(B), (C) and (D) out of which ONLY ONE is correct.

7. Section 7 contains 43 questions. The answer to each question is a single-digit integer, ranging from 0 to 9 (both inclusive).
8. Section 8 contains 17 questions. The answer to each question is a double-digit integer, ranging from 00 to 99 (both inclusive).

B. Marking Scheme

9. For each question in Section 1, you will be awarded 3 marks if you darken the bubble corresponding to only the correct answer
and zero mark if no bubbles are darkened. In all other cases, minus one (–1) mark will be awarded.

10. For each question in Section 2, you will be awarded 4 marks if you darken the bubble(s) corresponding to only the correct
answer and zero mark if no bubbles are darkened. No negative marks will be awarded for incorrect answers in this section.

11. For each question in Section 3, you will be awarded 4 marks if you darken all the bubble(s) corresponding to only the correct
answer(s) and zero mark if no bubbles are darkened.

12. For each question in Section 4 contains 10 paragraphs each describing theory, experiment, data etc. Twenty questions
related to Ten paragraphs with one or two or three questions on each paragraph. Each question of a paragraph has ONLY ONE
correct answer among the four choices (A), (B), (C) and (D).

13. For each question in Section–5, you will be awarded 2 marks for each row in which you have darkened the bubble
corresponding to the correct answer. Thus, each question in this section carries a maximum of 8 marks. There is no negative
marking for incorrect answer(s) in this section.

14. For each question in Section 6, you will be awarded 3 marks if you darken all the bubble(s) corresponding to only the correct
answer(s) and zero mark if no bubbles are darkened. In all other cases, minus one (–1) mark will be awarded.

15. For each question in Section 7, you will be awarded 4 marks if you darken the bubble corresponding to only the correct answer
and zero mark if no bubbles are darkened. No negative marks will be awarded for incorrect answers in this section.

16. For each question in Section 8, you will be awarded 4 marks if you darken all the bubble(s) corresponding to only the correct
answer(s) and zero mark if no bubbles are darkened. No negative marks will be awarded for incorrect answers in this section.
MATHEMATICS

SECTION – 1 : (Only One option correct Type)

This section contains 53 multiple choice questions. Each question has four choices (A), (B), (C) and (D)
out of which ONLY ONE is correct.

1. Let f(x) = sin2(x + ) + sin2(x + ) – 2cos( – ) sin(x + ) sin(x + ). Which of the following is TRUE?
(A) f(x) is strictly increasing in x  (, )
(B) f(x) is strictly decreasing in x  (, )
   
(C) f(x) is strictly increasing in x   ,  and strictly decreasing in x   , 
 2   2 
(D) f(x) is a constant function

2. If the roots of the equation a(b – c)x2 + b(c – a)x + c(a – b) = 0 (where a, b, c are unequal real numbers) are
real and equal and  be the roots of equation ax2 + bx + c = 0, a  0 then harmonic mean of  is
(A) 1 –  (B) 1 +  (C)  – 1 (D) –1 – 

1 2 a b 3a  3d
3. If A =   and B =   are two matrices such that AB = BA and c  0, then value of 3b  c
3 4  c d 
is
(A) 0 (B) 2 (C) –2 (D) –1

x
e(x -1)
- xx
lim+
4. x 0 x 2 =
(x²) -1
1
(A) 1 (B) (C) –1 (D) does not exist
8

5. If the equation 4y3 – 8a2yx2 – 3ay2x + 8x3 = 0 represent three straight lines, two of them are perpendicular
then sum of all possible values of a is
3 4
(A) – (B) 5 (C) (D) – 3
4 5
6. If p th , qth , r th terms of an AP are P, Q, R respectively , then which of the following must be CORRECT ?
(A) p Q + q R + r P = p R + r Q + q P
(B) (p + q + r)th term will be P + Q + R.
(C) If p < q < r, then P < Q < R
(D) If P,Q,R N then common difference of AP will be an integer..

7. The set of values of 'a' for which x2 + ax + sin–1(x2 – 4x + 5) + cos–1 (x2 – 4x + 5) = 0 has at least one solution
is
(A) (– , – 2 ]  [ 2 , ) (B) (– , – 2 )  ( 2 , )

8
(C) R (D) –  
 4 

8. Let  = log10 15 and  = log10 16 , a set A = {log 101, log10 2,.........log10 50} the number of elements in set
A which can be written in the form of a  b  c where a,b,c are rational numbers, is
(A) 23 (B) 24 (C) 25 (D) none of these

9. If both the roots of the equation x2 + (3 – 2k) x – 6k = 0 belongs to the interval (–6, 10), then largest value of
k is
(A) –1 (B) 3 (C) 5 (D) none of these

RESONANCE Page - 1
MATHEMATICS
(2x  1)
10. The domain of the function f(x) =
(2x  3 x 2  x )
3 + sin 1(log 2 x ) is

1  1 
(A)  ,   (B)  , 2 (C) [1, 2] (D) (1, )
2  2 

11. Let G, S, I be respectively centroid, circumcentre, incentre of triangle ABC. If R, r are circumradius and
inradius respectively then which of the following is INCORRECT ?
(A) SI2 = R2 (1 – cosA cosB cosC) ; A,B,C being angles of triangle.
(B) SI2 = R2 – 2Rr
1 2
(C) SG2 = R2 – (a + b2 + c2) ; a, b,c being sides of triangle.
9
(D) SG  SI

12. Two circles are given as x² + y² + 14x –6y + 40 = 0 and x² + y² – 2x + 6y + 7 = 0 with their centres as C1
and C2. If equation of another circle whose centre C3 lies on the line 3x + 4y –16 = 0 and touches the circle
C1 externally and also C1C2 + C2C3 + C3C1 is minimum, is x² + y² + ax + by + c = 0 then the value of
( a + b + c) is
(A) 2 (B) 0 (C) 16 (D) None of these

    
[f(x)], x   0,    ,  
13. If g(x) =   2   2  where [x] denotes the greatest integer function and
 3, x  /2

2(sin x  sinn x) | sin x  sinn x |


f(x) = ,n  R – {1}, then
2(sin x  sinn x) | sin x  sinn x |
(A) g(x) is continuous and differentiable at x =  /2, when 0 < n < 1
(B) g(x) is continuous and differentiable at x =  /2, when n > 1
(C) g(x) is continuous but not differentiable at x =  /2, when 0 < n < 1
(D) g(x) is neither continuous nor differentiable, at x =  /2, when n > 1

sec 2 x  2010 P( x ) 


14. If  2010 dx = 2010 + C, then value of P  is
sin x (sin x ) 3
1 3 3
(A) 0 (B) (C) 3 (D)
3 2

15. If î  ĵ bisects the angle between c and j  k , then value of c.j


  is equal to

1 1
(A) 0 (B) (C)  (D) 1
2 2

16. If A and B are two square matrices such that B = – A–1 BA, then (A + B)2 is equal to
(A) O (B) A2 + B2 (C) A2 + 2AB + B2 (D) A + B

2 3 12
17. The value of  where  = sin–1 + cos –1 + sec –1 ( 2 ) is equal to
4 4

  
(A) 0 (B) (C) (D)
4 6 2
 x , xQ
18. f(x) =  . The number of points of continuity of f(x) is
cos x , x  Q
(A) 0 (B) 1 (C) 2 (D) infinitely many

RESONANCE Page - 2
MATHEMATICS
2 2
dy
19. Let f(x) = 
x 1  y3
. The value of the integral  xf(x) dx is equal to
0

4 2 1
(A) 1 (B) (C) (D)
3 3 3


20. Let P be a point on ellipse 4x² + y² = 8 with eccentric angle . If tangent at P intersects the x-axis at A and
4
y-axis at B and normal at P intersect the x-axis at A' and y-axis at B'. The ratio of area of triangle APA' to
area of triangleBPB' is
(A) 1 : 1 (B) 2 : 1 (C) 3 : 1 (D) 4 : 1

1  sin x  5 9  
21.  1  cos x e–x/2 dx, x   , 
 2 2 

x x x x
(A) e–x/2 sin +C (B) – e–x/2 sec +C (C) –e–x/2 sin +C (D) –e–x/2 cos +C
2 2 2 2

e [ x ]  |x| – 2
22. lim = (where [.] denotes greatest integer function)
x 0 – [ x ] | x |

1 1
(A) 2 – (B) –1 (C) 2 – e (D) –2
e e

1 1 1 1  1  1  1 
23. If   tan1(1)  tan1(2)  tan 1(3) ,   tan (1)  2tan    3tan   and
2 3 2 3

p r
    cot 1(3) ,
q s
where p, q, r, s  N and are in their lowest form then which of the following is INCORRECT ?
(A) p  r  0 (B) q  4s (C) p  q  r  s  42 (D) pr  1  q

24. The length of sub-normal at any point P(x,y) on the curve, which (curve) is passing through Q(0, 1) is unity.
The area bounded by the curves satisfying this condition is equal to
2 4 1 8
(A) (B) (C) (D)
3 3 3 3

25. Let f(x) = x + x² + x4 + x8 + x16 + x32 + ......the coefficient of x10 in f(f(x)) is


(A) 28 (B) 40 (C) 52 (D) none of these
2008

26. If 1, 1, 2, 3,.....2008 are (2009)th roots of unity, then the value of  r(
r 1
r   2009–r ) is equal to

(A) 2009 (B) 2008 (C) 0 (D) – 2009

(cos x ) . sin(sec x )
27. Let lim 2
  , where , R
x  
2 x – 
 2

7 5
(A)  = 2,  = 0 (B)  = 0,  = 2 (C)  = ,=0 (D)  = ,=0
3 3

RESONANCE Page - 3
MATHEMATICS
28. Let A(x1, y1), B(x2, y2) C(x3, y3) be vertices of triangle ABC with BC = a, AB = c, AC = b. If algebraic sum of
 3ax 1 3ay 1   3bx 2 3by 2   3cx 3 3cy 3 
perpendicular distances from L  ,  , M ,  , N ,  to a
 a  b  c a  b  c   a  b  c a  b  c   a  b  c a bc
variable line is zero then all such lines passes through
(A) orthocentre of ABC (B) centroid of ABC
(C) circumcentre of ABC (D) incentre of ABC

29. If |f(x)|  1  x  R and f(0) = 0 = f(0) then which of the following can be TRUE?
 1 1  1 1 1
(A) f    (B) f  –   (C) f – 3   (D) none of these
3 5  3 4 3

30. If f(x) = cos 8{x} + sin 2x cosec 2x (where {.} represents fractional part function), then fundamental period
of f(x) is
1 1 7
(A) (B) (C) 1 (D)
4 2 8

31. The principle argument of z = x + iy, if it lies in second quadrant is equal to


 y y y  y
(A)  tan –1 (B)  – tan
–1
(C)   tan
–1
(D) – tan –1
2 x x x 2 x

 x 2 {e1/ x } , x  0
32. If the function f(x) =  , x  0 where {.} denotes fractional part function, is continuous at x = 0, then
 k
(A) k = 1 (B) f(x) is non-derivable at x = 0
(C) f(x) is derivable at x = 0 (D) f(x) is continuous at every point in its domain.

33. f(x) = – x2 + 1, g(x) = – 3


x . Then (gofogofogogog)(x) is
(A) odd function (B) even function (C) polynomial function (D) identity function

34. If a = 2 , then sum of series


cot–1(2a–1 + a) + cot–1 (2a–1 + 3a) + cot–1(2a–1 + 6a) + cot–1 (2a–1 + 10a) + .... upto infinite terms, is
   
(A) (B) (C) (D)
4 2 3 6

35. The normal to the curve 5x5 – 10x3 + x + 2y + 6 = 0 at P (0, –3) meets the curve again at two points, then the
equations of the tangents to the curve at these points is/are
(A) 2x – y – 3 = 0 (B) 2x + y – 3 = 0 (C) 2x + y + 3 = 0 (D) 2x – y + 3 = 0

2
36. If f(x + y + 1) =  f (x)  f (y)  and f(0) = 1, x, y  R then f(x) can be
2
(A) 1 – x (B) 1 – x (C) (x + 1)2 (D) x2 – 1

37. f(x) = cos –1(e 2 x  sin –1 x )


(A) domain of f is [–1, 1] (B) f is injective
(C) range of f is [0, 5] (D) f –1(x) = f(x) have no solution.

38. A normal chord drawn to a parabola y2 = 4x at any point P intersects the parabola again at a point Q, then the
minimum distance of Q from the vertex of the parabola is equal to
(A) 4 2 (B) 4 3 (C) 4 5 (D) 4 6

39. Let p(x) be a real polynomial of least degree which has a local maximum at x = 2 and a local minimum at
x = 4. If p(2) = 8, p(4) = 1 , then p(0) is
168
(A) (B) 42 (C) 43 (D) 45
5

RESONANCE Page - 4
MATHEMATICS
2
40. Let f(x) = (3x + 2)2 – 1, – < x  – . If g(x) is the function whose graph is the reflection of the graph of f(x)
3
with respect to line y = x, then g(x) equals to
1 1
(A) (–2  x  1), x  –1 (B) (–2 – x  1), x  –1
3 3
1 1
(C) (–1– x  2 ), x  –2 (D) (–1  x  2 ), x  –2
3 3

9  33   129 
41. cot–1   + cot–1   + cot–1   ............  is
2  4   8 

  
(A) (B) (C)  (D) none of these
4 2 4
 100   99   98  3 2 n
42. The sum  +   +   +.......+   +   is equal to, where   = n Cr
 98   97   96   1 0 r 

 100   101  100   101


(A)   (B)   (C)   (D)  
 98   99   99   98 

43. A circle with centre (3, 3) and of variable radius cuts the hyperbola x2 – y2 = 36 at the points P,Q,R and S.
If the locus of centroid of PQR is (x – 2)2 – (y – 2)2 = , then the value of  is
(A) 3 (B) – 2 (C) 4 (D) – 3

4 x – x 3  n(b 2 – 3b  3) , 2  x  3
44. Let f(x) =  . Find all the possible real values of b such that f(x) has the
 x – 18 , x3
smallest value at x = 3.
(A) (–, 2]  [3, ) (B) (–, 1]  [2, ) (C) (1, 2] (D) (–, 2]

 
45. The solution set of the inequality (cosec–1x)2 – 2(cosec–1x)  cosec–1x – is (–, a]  [b, ), then
6 3
(a + b) is equal to
(A) 0 (B) 2 (C) – 3 (D) 1

 1+ Z1 + Z 2 + Z3  ......... + Z7 
46. arg  =
 1+ Z8 + Z9 + Z10 + ....... + Z14 
1
(A) arg (Z1 Z2 .........Z7) (B) arg (Z1 Z2 .........Z7)
2
1 1
(C) arg (Z1 Z2 .........Z7) (D) arg (Z1 Z2 .........Z7)
4 8

 2  2 
47. Image of the curve xy = 1 in the curve  x  x  1   y  y  1  = 1 (on coordinate plane) is
  
(A) xy = 1 (B) xy + 1 = 0 (C) xy = 0 (D) x2 + y2 = 1

48. |1 + Z1 + Z2 +.....+ Z7 | =
 7  1  7   7 
(A) sec   (B) sec   (C) 2 sec   (D) None of these
 15  2  15   15 

49. Let O be centre, S, S be foci of hyperbola. If tangent at any point P on hyperbola cuts asymptotes at M and
N then OM + ON =
(A) |SP – SP| (B) SP + SP (C) SS (D) distance between vertices

RESONANCE Page - 5
MATHEMATICS
50. DABC be a tetrahedron such that AD is perpendicular to the base ABC and ABC = 30º. The volume of
tetrahedron is 18. If value of AB + BC + AD is minimum then length of AC is

(A) 3  6 3  (B) 3  6 2  (C) 2  6 3  (D) 2  6 2 


 1– x2 
51. The solution set of values of x satisfying equation 2cot–1x + cos–1     , is
 1 x2 
 
(A) all real numbers (B) (–, 0] (C) [0, ) (D) (–, –1)  (1, )

52. If g(x) = 2f (2x3 – 3x2) + f (6x2 – 4x3 – 3),  x  R and f  (x) > 0,  x  R, then g(x) is increasing on the
interval
 1  1 
(A)  ,     0,1 (B)   ,0   1,   (C)  0,  (D)  ,1
 2  2 

53. Let a circle with centre at C be made to pass through the point P(1,2), touching the straight lines 7x – y = 5
and x + y + 13 = 0 at A and B respectively. If tangents at A and B intersect at Q, then

(A) area of quadrilateral ACBQ can be 100 sq. units (B) radius of circle can be 40
(C) area of quadrilateral ACBQ can be 200 sq. units (D) radius of circle can be10

SECTION – 2 : (One or more options correct Type)


This section contains 37 multiple choice questions. Each question has four choices (A), (B), (C) and (D)
out of which ONE or MORE are correct.

54. Consider the locus of the complex number z in the Argand plane is given by Re(z) –2 = |z – 7 + 2i|. Let P(z1)
and Q (z2) be two complex number satisfying the given locus and also satisfying
 z - (2 + i)  
arg  1  = (  R) then the minimum value of PQ is divisible by
 z 2 - (2 + i)  2
(A) 3 (B) 5 (C) 7 (D) 2

55. Let f(x2 + y) = (f(x))2 + f(y) for all x, y  R, then


(A) f(x) is odd
(B) f(x) is even  f(x) = 0  x  R
(C) f(x) is continuous at x = 0  it is continuous everywhere
(D) f(x) is differentiable at x = 0  f(x) = x f (0),  x  R

56. Let f(x) = x3 + px2 + qx + r, where p,q and r are integers, f(0) and f(–1) are odd integers. Which of the
following is/are CORRECT ?
(A) f(1) is an even integer (B) f(1) is an odd integer
(C) f(x) = 0 has three distinct integer roots (D) f(x) = 0 cannot have three integer roots.

57. Let a,b,c  R. If ax 2 + bx + c = 0 have one root < –1 and other root > 1, then
b c b c b c b c
(A) 1 –  0 (B) 1 +  0 (C) 1 –  0 (D) 1 –  0
a a a a a a a a

1 log  x  5 
58. The value(s) of x satisfying 1 – log9(x + 1)2 = 3
  is/are
2 x3
(A) 1 (B) –2 (C) –7 (D) –4

x m f ( x )  h( x )  3
59. If g(x) = mlim when x  1 and g(1) = e3 such that f(x), g(x) and h(x) are continuous

2x m  4 x  1
functions at x = 1, then
(A) f(1) = 2e3 (B) h(1) = 5e3 – 3 (C) f(1) + h(1) = 7e3 + 5 (D) f(1) – h (1) = 7e3 + 5

RESONANCE Page - 6
MATHEMATICS
60. A parabola C whose focus is S(0,0) and passing through P(3, 4). Equation of tangent at P to parabola is
3x + 4y – 25 = 0. A chord through S parallel to tangent at P intersects the parabola at A and B. Which of the
following are CORRECT ?
(A) Length of AB is 20 units
(B) Latus rectum of parabola is 20 units
(C) Only one real normal can be drawn from the point (–3, –4)
(D) Only one real normal can be drawn from the point (–6, –8)

61. If  11  , where a and b are natural numbers then b is divisible by


4 + 8 - 32 + 768 = a 2 cos  
 b  a
(A) 2 (B) 4 (C) 3 (D) 6

62. For the function f(x) = (x2 + bx + c) ex and g(x) = (x2 + bx + c) ex + ex (2x + b), which of the following holds
good ?
(A) f(x) > 0 for all real x g(x) > 0 (B) f(x) > 0 for all real x  g(x) > 0
(C) g(x) > 0 for all real x  f(x) > 0 (D) g(x) > 0 for all real x  f(x) > 0
2 
 1   1 
 1   cos1 x 
63. If the equation 2 1 x
 cos   a   2  a2  0 has only one real solution then subsets of values
   2  
   
of 'a' are
(A) (–3, 1) (B) (–, –3] (C) [1, ) (D) [–3, )

 x2 , x 2
 2
64. f(x) =  x , 2x3
 3 , x3
 x – 18
(A) f(x) is continuous in R. (B) f(x) is discontinuous at 'a' for some a  R.
(C) f(x) is continuous at infinitely many real x. (D) f(x) is discontinuous at infinitely many real x.
15
 4 1 
65. Which of the following must hold good for the expansion of the binomial  x  3  ?
 x 
(A) There exist a term which is independent of x.
(B) 8th and 9th terms of the expansion have the greatest binomial coefficient
(C) Coefficients of x32 and x–17 are equal
(D) If x = 2 , then number of rational terms in the expansion is 5

66. Which of the following is NOT in the solution set of the inequality |x 2 – 1| + |2x 2 – 3| < |2 – x 2| ?
 1
(A) (–1, 1) (B) (–2, 2) (C)  – 1 ,  (D) (3, )
 2

1 1
67. Let a function f(x), x  0 be such that f (x)  f    f (x)  f   then f(x) can be :
x
  x

(A) 1 – x2013 (B) | x | 1

 2
(C) (D) , k being arbitrary constant
2 tan 1 | x | 1  k n | x |
68. TP and TQ are tangents to parabola y2 = 8x and normals at P and Q intersect at a point R on the parabola.
The locus of circumcentre of TPQ is a parabola whose -
7 
(A) Vertex is (2, 0) (B)foot of perpendicular from focus on the directrix is  , 0 
4 

9 
(C) length of latus rectum is 1 (D) focus is  , 0 
4 

RESONANCE Page - 7
MATHEMATICS
69. The function f(x) = 2|x| + |x + 1| – ||x – 3| – 3|x|| has a local minimum or a local maximum at x =
3 3
(A) 0 (B) – (C) (D) 3
2 4

70. If z1 , z2 ,z 3 , z4 are roots of the equation a0z4 + a1z3 + a2z2 + a3z + a4 = 0, where a0 , a1 , a2 , a3 and a4 are real,
then
(A) z 1 , z 2 , z 3 , z 4 are also roots of the equation

(B) z1 is equal to at least one of z 1 , z 2 , z 3 , z 4

(C) – z 1 , – z 2 ,– z 3 ,– z 4 are also roots of the equation


(D) None of the above.

sin(e x–3 – 1)
71. lim = c, which of the following is/are CORRECT ?
X 3 n(x – 2)


(A) sin–1c = – (B) c = 0
2
(C) c is a positive integer (D) c is neither prime nor composite

     
72. Let a, b and c be three unit vectors such that | a  b  c | = 3 and
           
(a  b).(b  c )  (b  c ).(c  a)  (c  a).(a  b )   . Which of the following are CORRECT ?

(A) The maximum value of  is 0


  
(B) If  is maximum then the volume of parallelepiped determined by a, b and c is 3
        
(C) If  is maximum then the value of (2a  3b  4c ).(a  b  5b  c  6c  a) is 32.

(D) None of these

73. Let g be the inverse of the continuous function f, Let there be a point (, ), where , is such that it
satisfies each of y = f(x) and y = g(x) then
(A) the equation f(x) = g(x) has infinitely many solutions
(B) the equation f(x) = g(x) has atleast 3 solutions
(C) f must be a decreasing function of x
(D) g can be an increasing function of x

74. The maximum and minimum value of ab sin x + b 1 a 2 cos x + c (|a| <1,b > 0), lie in interval
(A) [c – b, b + c] (B) (b – c, b + c)
(C) [c – 2b, c + 2b] (D) [–c, c]

2 log ( x 1) 3
75. If lim (log 3 (ax  3x  1))   , where  is a finite real number then
x2

(A) a = – 1 (B) 'a' can have more than one values


(C)  = e–2/3 (D)  = e–1/3

76. Which of the following functions is an injective (one-one) function in their respective domain?
(A) f(x) = 2x + sin 3x (B) f(x) = x. [x],(where[.] denotes the G.I.F.)
2x +1
(C) f(x) = (D) None of these
4 x -1

RESONANCE Page - 8
MATHEMATICS
77. The centre of a circle S = 0 lies on 2x – 2y + 9 = 0 and S = 0 cuts orthogonally the circle x2 + y2 = 4. Then
the circle must pass through the point
(A) (1, 1) (B) (– 1/2, 1/2) (C) (5, 5) (D) (– 4, 4)a

 1   1   x  
78. If 2f(x) + xf   - 2f  2sin  x +   = 4cos²   + xcos ,  x  R - {0} then which of the following
x
    4   2
  x
statements(s) is/are true?
 1
(A) f(2) + f   = 1 (B) f(2) + f(1) = 0
2

 1  1
(C) f(2) + f(1) = f   (D) f(1) + f   = 1
2 2

79. If  11  , where a and b are natural numbers then (b – a) is divisible by


4 + 8 - 32 + 768 = a 2 cos  
 b 
(A) 2 (B) 23 (C) 69 (D) 46

 
80. Equation of the plane passing through the line of intersection of the two planes r . n1  q1 and r . n2  q2 and
 
parallel to the line of intersection of r . n3  q3 and r . n4  q4 is
   
(A) dependent on n1 . n 3 (B) dependent on n3 . n4
(C) independent of q1 and q2 (D) independent of q3 and q4

x 1 y  2 z  3
81. Let ABCD be a tetrahedron, where A = (2,0,0), B = (0,4,0). If edge CD lies on line   , such
1 2 3

5
2 –
that centriod (,,) of tetrahedron satisfies 2   – y1   – z1 then which of the following are
1 a b
CORRECT?

5 19 1
(A) a + b = (B) y1 + z1 = (C) y1 – z1 = (D) a + b + y1 = 5
2 4 4

82. The line 3x + 6y = k intersects the curve 2x2 + 2xy + 3y2 = 1 at points A and B. The circle on AB as diameter
passes through the origin. The possible value of k is -
(A) 3 (B) 4 (C) – 4 (D) –3

83. A rod of length 2 units whose one end is (1, 0, –1) and other end touches the plane x – 2y + 2z + 4 = 0, is
rotated on this plane, then
(A) the rod sweeps a solid structure whose volume is  cubic units
(B) the area of the region which the rod traces on the plane is 2
(C) the length of projection of the rod on the plane is 3 units
2 2 5
(D) the centre of the region which the rod traces on the plane is  , , 
3 3 3 

  
84. Let f : 0,   [0, 1] be a differentiable function such that f(0) = 0, f   = 1, then
 2  2

  2  
(A) f () = 1  ( f ( ))2 for all    0,  (B) f () = for all    0, 
 2   2

1   8  
(C) f() f () = for at least one    0,  (D) f () = for at least one    0, 
  2 2  2

RESONANCE Page - 9
MATHEMATICS
85. Two equal sides of an isosceles triangle are given by the equation 7x – y + 3 = 0 and x + y – 3 = 0 and its third
side passes through the point (1, – 10). The equation of the third side can be
(A) x + 3y + 29 = 0 (B) x – 3y = 31 (C) 3x + y = 3 (D) 3x + y + 7 = 0

86. Let f(x) = cos–1 (cos2x) and g(x) = |cos x| then


(A) number of solution of f(x) = g(x) in [0,2] is 4.
(B) max {f(x), g(x)} is a periodic function
(C) max {f(x), g(x)} is a non differentiable function for some x,
(D) min {f(x), g(x)} is an even function

 3 
87. Consider f(x) = 2sinx + sin 2x , x 0,
 2 

3 3
(A) Greatest value of f(x) is 3 (B) Greatest value of f(x) is
2
(C) Least value of f(x) is zero (D) Least value of f(x) is – 2

88. For a Function f : A  B such that n(A)  a, n(B)  b (a,bN) then which of the following statements
s
must be CORRECT ?
(A) If function is one - one, onto, then a>b
(B) If function is one - one, into, then a<b
(C) If function is many - one, onto, then a>b
(D) If function is many - one, into, then a<b

89. Which of the following statement(s) is/are 'CORRECT’ ?


(A) Rolle’s theorem is applicable to the function f(x) = 1 – 5
x 6 in the interval [–1, 1]

x
t2

(B) The value of xlim


 xe
0
dt
is equal to – 2
0
x
1 x – e

1 1
 
(C) If a and b are any two unit vectors, then least value of   2 +   2 is equal to 2.
| a b | | a– b |


(D) The angle between pair of tangents drawn to the curve 7x2 – 12y2 = 84 from M(1, 2) is
4

90. If a is an integer and m 1, m 2, m 3 are the slopes of all three straight lines represented by equation
y3 + (2a + 5) xy2 – 6x2y – 2ax3 = 0 which are also integers, then which of the following can holds good?
3 3

(A) a +  mi = – 1 (B) a + m i =–5


i 1 i 1

3 3

(C) a +  mi = 0 (D) a +  m = 32 i
i 1 i 1

91. If fog(x) = 1 – x,  x  R and fof(x) = 1 + sin(f(x)),  x  R, then


(A) f(0) = 1 (B) f() = 1 (C) f(x) is periodic (D) f(x)  0,  x  R

RESONANCE Page - 10
MATHEMATICS

SECTION - 3 (True & False Statement Type)


This Section Contains 2 questions.Each questions contains 4 statements S1, S2, S 3 & S4. Each
statement is either true (T) or false (F). Each questions has 4 choices (A), (B), (C) and (D) each of which
contains whether S1, S2, S3 & S4 are true or false. Exactly one choice contains the correct order of truthness
or falseness of S1, S2, S3 & S4 respectively and is the correct choice.

92. Consider the following statements :


S1 : Let ax2 + bx + c = 0 be a quadratic equation, if b2 – 4ac > 0 then roots are real and distinct
S2 : Let a,b,c,d . Then the possible rational roots of the equation ax3 + bx2 + cx + d = 0 are of the
p
form where p is an integral divisor of d and q is an integral divisor of a.
q
S3 : Let z = a + ib be a complex numbers where a,b  C – R. Then z  a – ib .

3– 5
S4 : If (a  b 5 )2 = , then a,b  Q.
2
State, in order, whether S1, S2, S3, S4 are true or false
(A) FTTT (B) TFTT (C) FTTF (D) FFTF

93. Consider the following statements :


S1 : Let a,b,c  C and ax2 + bx + c = 0 be a quadratic equation. Then b2 – 4ac = 0
 roots are real and equal.

S2 : Let b,c  I and b2 – 4c be a perfect square. Then roots of the equation x2 + bx + c = 0 may not be
integers.
S3 : If the quadratic equations a1x2 + b1x + c1 = 0 and a2x2 + b2x + c2 = 0 have common root, then
a1 b1 c1
 
a 2 b2 c 2 .

a1x 2  b1x  c 1 x  m
S4 : f(x) = 2 , g(x) = where a1, a2,  , a are non zero real and other coefficients are
a2 x  b2 x  c 2 ax  b
also real. Then range of f(x)  range of g(x).
State, in order, whether S1, S2, S3, S4 are true or false
(A) TTFT (B) TTTF (C) FFTT (D) FFFF

SECTION – 4 : (Paragraph Type)


This section contains 10 paragraphs each describing theory, experiment, data etc. twenty questions
relate to two paragraphs with one or two or three questions on each paragraph. Each question of a
paragraph has only one correct answer among the four choices (A), (B), (C) and (D).

Paragraph for Question Nos. 94


x2 y2
Let  = 1 be an ellipse and P be any point on it and S and S be its foci.
36 27

94. The point of intersection of straight lines joining each focus to the foot of the perpendicular from the other

focus upon the tangent at a point P   , is
3

 21 9   15 9   15 9   21 9 
(A)  ,  (B)  ,  (C)  ,  (D)  , 
 8 4  8 4  4 8  4 8

RESONANCE Page - 11
MATHEMATICS
Paragraph for Question Nos. 95 to 96
Consider f(x) = log{x}[x] where [.] is the greatest integer function and {x} is fractional part function.

95. f(x) is
(A) a one-one function (B) a many-one function
(C) a odd function (D) a periodic function

96. Consider the inequality f(x) < 2. Number of solutions of this inequality in x (–1, 2) is/are
(A) 0 (B) 1 (C) 3 (D) infinitely many

Paragraph for Question Nos. 97 to 98

Consider a polynomial y = P(x) of the least degree passing through A(–1, 1) and whose graph has two points
of inflection B(1, 1) and C with abscissa 0 at which the curve is inclined to the positive direction of
x-axis at an angle of sec–1( 2 ).

97. The value of P(0) is


1 1
(A) – (B) (C) 0 (D) 2
2 2

98. The equation P(x) = 0 has


(A) three distinct real roots
(B) one real root
(C) three real roots such that one root is repeated.
(D) none of these

Paragraph for Question Nos. 99 to 100


x y
If cos –1 + cos –1 =  then the value of 9x 2 – 12xy cos + 4y 2 is equal to N sin 2 
2 3
(where N is a positive integer) and complete set of values of x for which (cos–1 x)2 – (sin–1 x)2 > 0 is satisfied,
is [p, q).

99. Which of the following values lie in [p, q) ?


(A) N –3 (B) N –4 (C) N –5 (D) N –6

100. Which of the following functions cannot be defined at x = N – 6 ?


(A) sin–1 x (B) cos–1 x (C) tan–1 x (D) sec–1 x

Paragraph for Question Nos. 101 to 102

Base of a pyramid is rectangular, three of its vertices of the base are A(2,2, –1), B(3,1,2) and C(1,1,1) and its

 –26 –10 
vertex at the top is P  4, , .Fourth vertex of the base is D. Then answer the following questions.
 3 3 

101. If the coordinates of foot of normal drawn from P on the base of the pyramid are (  , m,n), then
(A)  + m + n = 4 (B)  = 3m (C) n = 3m (D)  < m < n

102. Volume (in cubic unit) of the pyramid is


(A) 20 (B) 30 (C) 20 (D) 30

RESONANCE Page - 12
MATHEMATICS
Paragraph for Question Nos. 103 to 104

 ( 12 – 2) x 2 
Consider f(x) = tan–1  4 2
 and m and M are respectively minimum and maximum values of f(x) and
 x  2x  3 
x = a (a > 0) is the point in the domain of f(x), where f(x) attains its maximum value.

 –1  7M  –1  3  
103. If cos–1x + cos–1y = 3  tan  tan   tan  m  tan 8   , then (x + y) is equal to
  2   
(A) 2 (B) – 2 (C) 0 (D) 3/2

104. If  and  are roots of the equation x2 – (tan(3sin–1 (sinM))) x + a4 = 0, then – ( + ) equals to
(A) 1 (B) 4 (C) 3 (D) 2

Paragraph for Question Nos. 105 to 106

Consider the quadratic equation ax2 + (2 – a)x – 2 = 0, where a  R.

105. If exactly one root is negative, then the range of a2 + 2a + 5 is


(A) [4, ) (B) [–2, ) (C) (–, 4] (D) (5, )

106. If ,  are roots of the quadratic equation and if there are at least four negative integers between  and ,
then the complete set of values of a is
 7   1  3 1 7 
(A)  – ,–3  (B)  0,  (C)  – ,–  (D)  , 3 
 2   2  2 2 2 

Paragraph for Question Nos. 107 to 108

Let M be the maximum value of c2 for which O(0, 0) and A(1, 1) does not lie on opposite side of the straight
line (a + b)2x – (ab + bc + ca + 1)y + 2 = 0 for all a,b  R.
Also lx + my + n = 0 be a variable line, where l, m,n are 1st, 3rd and 7th terms of an arithmetic progression and
the variable straight line always passes through a fixed point P(,).

107. If the circles x2 + y2 = M + 1 and x2 + y2 – 24x – 10y + 2 = 0 have exactly two common tangents, then the
number of possible integral values of , is
(A) 11 (B) 13 (C) 12 (D) 10

108. The tangent of the circle x2 + y2 = M + 2 at the point ( – 1,  + 1) also touches the circle
x2 + y2 – 4x – 2y + 20 = 0 then its point of contact is
(A) (3, – 1) (B) (–3,0) (C) (–1, – 1) (D) (–2, 1)

Paragraph for Question Nos. 109 to 110

Let f(x) = x 2 + b1x + c 1 , g(x) = x 2 + b2x + c 2 , ,  be real roots of f(x) = 0 and  +  ,  +  be real roots
1
of g(x) = 0, minimum value of y = f(x) be – and minimum value of y = g(x) occurs at
4
7
x= .
2

109. Minimum value of y = g(x) is


1 1 1
(A) –1 (B) – (C) – (D) –
2 4 3

110. The value of b2 is


(A) 6 (B) –7 (C) 8 (D) 7

RESONANCE Page - 13
MATHEMATICS
Paragraph for Question Nos. 111 to 113

1 2 0
 
2 1 0
There exists a matrix Q such that PQPT = N, where P =  .
0 0 1

Given N is a diagonal matrix of form N = diag.(n1, n2, n3) where n1, n2, n3 are three values of n satisfying the
equation det.(P – nI) = 0, n1< n2< n3.
[Note : I is an identity matrix of order 3×3]

111. The value of det.(adjN) is equal to


[Note : adj M denotes the adjoint of a square matrix M.]
1 1
(A) 4 (B) (C) (D) 9
4 9

112. If QT = Q + , then the value of  is equal to


–1
(A) –1 (B) 0 (C) 1 (D)
3

113. The trace of matrix P2012 is equal to


(A) 32011 + 2 (B) 32012 (C) 32012 + 2 (D) 32011

SECTION - 5
Matrix - Match Type

This section contains 4 questions. Each question contains statements given in two
columns, which have to be matched. The statements in Column-I are labelled A,
B, C and D, while the statements in Column-II are labelled p, q, r, s and t. Any
given statement in Column-I can have correct matching with ONE OR MORE
statement(s) in Column-II. The appropriate bubbles corresponding to the answers
to these questions have to be drakened as illustrated in the following example.
If the correct matches are A-p, s and t ; B-q and r; C-p and q; and D-s and t;
then the correct darkening of bubbles will look like the following :

114. Match the inequality in column-I with their complete solution set in column-II.

Column –  Column – 

(A) logsinx log3 log0.2 x < 0 (p) [–1, 1]

(e x  1)(2x  3)( x 2  x  2)
(B)  0 (q) [–3, 6)
(sin x  2) x( x  1)

 1 
(C) |2 – | [x] – 1| |  2, (r)  0, 
 125 
[.] represents greatest integer function.

 3 
(D) |sin–1 (3x – 4x3)|  (s) (–, –1)   ,  
2 2 
(t) 

RESONANCE Page - 14
MATHEMATICS
2x – 1 x 1
115. If f(x) = and g(x) = .
x–2 x
Column –  Column – 

(A) Domain of f(g(x)) is (p) (–, 1)  (1, 2)  (2, )

(B) Domain of g(f(x)) is (q) (–, 2)  (2, )

(C) Range of f(f(x)) is (r) (–, 0)  (0, 1)  (1, )

(D) Range of g(g(x)) is (s) (–, 1)  (2, )

(t) (–, 1/2)  (1/2, 2)  (2, )

116. Column-I Column-II

 x 
(A) If f : (1/2, 1)  [0, ) and f(x) = log e   , then f(x) is (p) one-one function
 1– x 

(B) If f : [0, 1]  [cos(sin1) + sin(cos1), 1 + sin1] and (q) many-one function

f(x) = cos(sinx) + sin(cosx), then f(x) is

 x – 2m  2 , x  –1
(C) Let f : R  R and f(x) =  , then for (r) into function
 mx – 4 , x  –1
all values of m  (6, ). The function f is

( x –  )( x – )
(D) Let f(x) = , where  <  <  < , then f(x) is (s) surjective function
( x –  )( x –  )

(t) invertible function

117. Column-I Column-II


2 2 20
 1  1 1
(A) an = 1  1   + 1  1   Then, the value of  an is (p) 2
 n   n  n 1

not divisible by
(q) 5

1 1 2
(C) Number of solutions satisfying the equation – = (r) 7
sin x sin 2x sin 4 x
in [0, 4] equals

 4 1  3x  4
(D) Integral multiple of solution of the equation cot  tan1 2
  = is (s) 4
 2r  3x  2
 r 1 

(t) 6

RESONANCE Page - 15
MATHEMATICS

SECTION – 6 : (Matching list Type)


This section contains 1 multiple choice questions. Each question has matching lists. The codes for the lists
have choice (A), (B), (C) and (D) out of which ONLY ONE is correct.

118. Match List I with List II and select the correct answer using the code given below the lists :
List - I List - II
 2 3 9 10 11
P. sin sin sin ........sin sin = 1.
21 21 21 21 21 210
 2 3 9 10 1
Q. sin sin sin ........sin sin = 2.
22 22 22 22 22 210

 2 3 9 10 11
R. cos cos cos ........cos cos = 3.
21 21 21 21 21 25

 2 3 9 10 21
S. cos cos cos ........cos cos = 4.
22 22 22 22 22 210
Codes:
P Q R S
(A) 3 4 4 2
(B) 4 1 2 1
(C) 2 3 1 4
(D) 4 3 1 3

SECTION – 7 : (Integer value correct Type)


This section contains 43 questions. The answer to each question is a single digit integer, ranging from
0 to 9 (both inclusive).

 x log( x  x 2  1)
 x  ne , n  I
 x  e 1
119. If f(x) =    (where [.] denotes the greatest integer function),
  e  2
 0 x0
then find the value of f(0) + f(1947) + f(–1947)

120. The equation of common tangent to the curve xy = 4 and x2 + y2 = 8, whose x and y intercepts are positive
is x + y = . Find the value of .

121. If function y = f(x) has only two points of discontinuities say x1, x2 , where x1, x2 < 0, then find the number
of discontinuities of y = f(|x|) .

122. Given equation |x2 – 5x + 4 + sinx| = |x2 – 5x + 4| + |sinx|, where 0  x  2 then find sum of all integral values
present in complete solution set of given equation.

 1
123. If f(x) + f 1 –  = 1 + x  x  R – {0, 1}. Then find the value of 4f(2).
 x

124. If |z1| = 2, (1 – i) z2 + (1 + i) z 2  8 2 , (z1, z2 are complex variables) then the minimum value of |z1 – z2| =
2
 x   x 
125. If  2   2   k  0 for all real x, then find sum of modulus of all integral values of
 x – 5x  9   x – 5x  9 
k in [–4, 7].

126. Find the number of integral values of 'a' for which the equation
(3sinx – 4cosx)2 – (a2 + a + 5)|3sinx – 4cosx| + a3 + 3a2 + 2a + 6 = 0 has a real solution.

RESONANCE Page - 16
MATHEMATICS
127. In a sequence of circles c1, c2, c3, ....,cn , the centres lie along positive x-axis with abscissa forming an A.P.
of first term unity and common difference 3. The radius of these circles are in G.P. with first term unity and
common ratio 2. If the tangents with slope m1 and m2 of c3 intersect each other at the centre of c5, then find
the value of 10|m1m2|.

128. In a meeting there are six ministers all speak exactly two languages. M1 speaks only L1 and L2, M2 speaks
only L2 and L3, M3 speaks only L3 and L4, M4 speaks only L4 and L2, M5 speaks only L4 and L1, M6 speaks only
L1 and L3. If two ministers are chosen at random, the probability that they speak common language is , then
find the value of 5.

129. If a,b and c are distinct real numbers, such that the quadratic expressions Q1(x) = ax 2 + bx + c,
Q2(x) = bx2 + cx + a and Q3(x) = cx2 + ax + b are always non-negative, then find the number of possible
a 2  b2  c 2
integers in the range of the expression y = .
ab  bc  ca

130. Find sum of all real solutions of the equation xlog5 + log2 = x + log(2x – 1).

131. Given f : R  R; f(x) = 2x³ – 3(k + 2)x² + 12kx – 7, – 4  k  6, k  I, then find the number of values of k for
f(x) to be invertible.
a 2
2
132. The real value of a for which the integral  e – ( x – 2 ) dx attains its maximum value is . Find the value of ||.
a–2

x3
133. If f(x) =  3 x 2  k 2 x – 10 is a many one function, find sum of all positive integral values of k.
3
2
– 2 x 1) sin2 x
134. Find the minimum value of e( 2 x .

135. If a is the number of onto functions on A = {a,b,c,d} and b is the numbers of one one functions on A, find
|a – b|.

 
136. Find number of solutions of the equation sin3x = cos4x in the interval 0,  .
 4

137. If sum of the series cot–1 (2.12) + cot–1 (2.22) + cot–1 (2.32) + .......... is equal to k, then find the value
of [k], where [.] denotes greatest integer function.

138. The function f(x) = x2 e–2x, x > 0. If  is maximum value of f(x) then find [ –1 ] .
(where [.] represents greater integer function)

139. Let A be any variable point on the x-axis and B be the point (2, 3). The perpendicular at A to the line AB meets
the y-axis at C. Locus of mid-point of the segment AC as A moves is ax2 – 2x + by = 0. Find the value of
a + b.

140. x 2 + (a  b) x + (1  a  b) = 0, a, b  R. Find the least integral value of ' a ' for which both roots of the
equation are real and unequal  b  R.

141. If a1, a2, a3, 5, 4, a6,a7,a8,a9 are in H.P. and


a1 a2 a3
5 4 a6
= then find the value of [] is, where [.] denotes G.I.F..
a7 a8 a9

sin( x   ) sin( x  ) sin( x   )  f(k)


k 1
142. If f(x) = cos( x   ) cos( x  ) cos( x   ) and f(9) = (0), then find the value of .

cos( –  ) cos(  –  ) cos( – )

RESONANCE Page - 17
MATHEMATICS
2
log10
143. Find number of solutions of the equation x  log10 x   2log10 x  1 (where [.] denotes greatest
 
integer function)

1
sin–1 x 2  n 
144. If the value of the definite integral  2 dx  (n  N), then value of   is
0 x – x 1 n  27 

      
145. If K r  r  a  b , where K is a non zero scalar and a, b are two given vectors. Then r will be
 
1   a.b    
2  2  Kb  a  n ( a  b)  , find the value of   m  n .
K |a|  Km 
 


146. Sum of all elements in range of f(x) = 16 – xC2x – 1 + 20 – 3xC4x – 5 is  then find the value of .
469
1
147. If z = ( i 3 – 1) , then find the value of (z – z2 + 2z3)(2 – z + z2).
2

sin3 –1 1
cos 2 4 3
148. If = 0, then find the number of values of  in [0, 2].
2 7 7

149. If 1 lies between the roots of the quadratic equation 3x2 – 3(sin ) x – 2 cos2 = 0 then find sum of all integral
values of  in [0, 2], where  is in radians.

 
150. The volume of the parallelepiped whose coterminous edges are represented by the vectors 3( b  c ),
   
2( a  b ) and 4( c  a ) where

  
a = (1 + sin) i + (cos) j + (sin2) k

  2    2    4  
b = sin     i + cos     j + sin  2  k
 3   3   3 

 2    2    4  
c = sin   – 3  i + cos   – 3  j + sin  2 – 3  k

     

 
is 18 cubic units then find number of values of  in the interval  0,  .
 2

sin x  2
151. Let f(x) = x (1 + xcosx. n x+ sinx)dx and f   =
2 4
then find the value |cos(f())|, where x > 0.

T  2 [ x]    3 [ x]  
152. Find if T is the fundamental period of f(x) = sin   +cos   , where [.] denotes greatest
2  3   2 
integer function.
 – 2
153. f(x) = x3 + 4x2 + x + 2 is monotonically decreasing in the largest possible interval  – 2,  . Then find
 3 
greatest value of .
 3x   
154. If range of f(x) = n  2 sin x  tan x –  1  x   ,  is [a, b], then find the value of [a + b], where [.]
   6 3
denotes greatest integer function.

155. Find numbers of values of k if – x 2 + kx + 2 < 0 for exactly two integers.

RESONANCE Page - 18
MATHEMATICS
156. Let z1, z2 and z3 be complex numbers such that |z1| = |z2| = |z3| = |z1 + z2+ z3| = 2. If |z1 – z2| = |z1 – z3| and
z2  z3. Then find the value of |z1 + z2| |z1 + z3|.
a
157. Following usual notations in a triangle ABC, if R(a + b) = c ab then r = . Find the value of  + µ.
 µ

 ( f (2))2 
158. Let a function f(x) be such that f''(x) = f'(x) + ex and f(0) = 0, f'(0) = 1, then find the value of n  .
 4 

159. If the area bounded by |y| = sin–1|x| and x = 1 is a( + b), then find the value of (a – b).

160. If z1, z2, z3  C satisfy the system of equations given by |z1| = |z2| = |z3| = 1, z1 + z2 + z3 = 1 and z1z2z3 = 1 such
that Im(z1) < Im(z2) < Im(z3), then find the value of [|z1 + z22 + z33 |], where [.] denotes the greatest integer
function
2
 x   x 
161. If  2   2   k  0 for all real x, then find sum of modulus of all integral values of
 x – 5x  9   x – 5x  9 
k in [–4, 7].

162. Line L is a tangent to a unit circle S at a point P . Point A and the circle S are on the same side of L and the
distance from A to L is 3 unit. Two tangents from point A intersect line L at the point B and C. Find the value
of (PB)(PC)

SECTION – 8 : (Integer value correct Type)


This section contains 17 questions. The answer to each question is a Two digit integer, ranging from
00 to 99 (both inclusive).

163. If x,y,z are non-negative integers such that 2(x3 + y3 + z3) = 3(x + y + z)2 then maximum value of x + y + z
is

164. If [] denotes the greatest integer function and let f be a function defined on the set of all non-negative
integers and taking values in the same set. Given that -
 x   f (x) 
(A) x – f(x) = 20   – 10   for all non negative integers
20
   10 
(B) 2008 < f(2010) < 2012
then find the number of possible values of f(2010).

165. Eccentricity of an ellipse of minimum area, circumscribing two circles, of equal radius, touching externally,
22
is . Find the value of µ.
µ

166. Tangent at a point P (other than origin) on the curve y=x3 meets the curve again at P1 . The tangent at P1
a re a o f  P P P
1 2 3
meets the curve again at P2 and so on. then find the ratio a re a o f  P P P
1 2

167. Let f : N  [0, ) be a function satisfying the following conditions :


(a) f(100) = 10,
1 1 1 1
(b) + + +......+ = f(n +1), for all non-negative integers n. Find
f (0)  f (1) f (1)  f (2) f (2)  f (3) f (n)  f (n  1)
the value of f(9801).

RESONANCE Page - 19
MATHEMATICS
 z2  
168. Complex number z satisfies arg (z  (3 + 3i)) =  and arg   then range of  such that no z
 z  2i  11
exists is [a ,b], then find the value of [ 10 |a+b|] where [.] represents the greatest integer function and arg
means principal value of argument.
 3x   
169. Range of f(x) = n  2 sin x  tan x   1 ,  x   ,  is [a, b], then find the value of 11[a + b].
   6 3
[•] represents greatest integer function.

1 1
170. A function g(x) is defined as g(x) = f(2x2 – 1) + f(1 – x2) and f (x) is an strictly increasing function then
4 2

 p   p 
g(x) is strictly increasing on the interval  – , 0   ,   then find the value of p2+q2.
 q   q 

(where p and q are coprime to each other)

sin 2x a  b cos x
171. If  (3  4 cos x) 3
dx   e where a, b, c, d are positive integers and e is arbitrary constant
c(3  4 cos x )d
then minimum value of a + b + c + d equal to

172. If f(x) = x 3 – x 2 + 100x + 1001, then find the number of correct statements among the following
 1   1 
(i) f(2000) > f(2001), (ii) f   > f  , (iii) f(x + 1) > f(x – 1), (iv) f(3x – 5) > f(3x)
 1999   2000 
173. The numbers 2,3,4,5,6,7,8 are to be placed, one per square, in the diagram shown so that the sum of the four
numbers in the horizontal row equals 21 and the sum of the four numbers in the vertical column also equals
21. In how many different ways can this be done ?

174. If the function f(x) = sin(n x) – cos(n x), where –  n x  , strictly increases in interval [e,e] and the
length of [e,e] is greatest possible, then find the value of – 5 cos (–).
cos 5 x  cos 4 x sin 2x
175. If  dx = – sin x + C, find a.
2 cos 3 x  1 a

176. If largest subset of (0, p ) at each point of which the function f(x) = 3 cos4 x + 10 cos3 x + 6 cos2 x – 3 , is
    2 
decreasing is  0,    ,   , then find the value of (p + r).
 p  r 
177. Consider seven digit number x1 x2.......x7, where x1, x2.......,x7,  0 having the property that x4 is the great-
est digit and all digits towards the left and right of x4 are in decreasing order. Then total number of such
number in which all digits are distinct is a, then find the sum of digits of a.
178. Let a  tan1 tan30,b  cos1 cos 40,c  sin1 sin(b  a) then product of all integral values of x satisfying

cos1 cos x  x  a  b  c is

3n 2  1

9
179. If S =  (n
n2
2
– 1)3
then
S
=

1
3n
 Cn  n A
180. If lim  2n  = , where A & B are relatively prime numbers, then A+B is equal to-
n
 Cn  B

RESONANCE Page - 20
MATHEMATICS

CRITICAL QUESTION BANK

ANSWER KEY
1. (D) 2. (D) 3. (D) 4. (B) 5. (A) 6. (A) 7. (D)

8. (B) 9. (D) 10. (C) 11. (A) 12. (B) 13. (B) 14. (C)

15. (A) 16. (B) 17. (D) 18. (B) 19. (C) 20. (D) 21. (B)

22. (A) 23. (C) 24. (B) 25. (B) 26. (D) 27. (C) 28. (D)

29. (C) 30. (B) 31. (B) 32. (C) 33. (B) 34. (A) 35. (A)

36. (C) 37. (B) 38. (D) 39. (B) 40. (B) 41. (D) 42. (D)

43. (C) 44. (B) 45. (D) 46. (C) 47. (A) 48. (B) 49. (B)

50. (B) 51. (C) 52. (B) 53. (A) 54. (BD) 55. (ABCD) 56. (BD)

57. (BD) 58. (ABC) 59. (AB) 60. (ABC) 61. (ABCD) 62. (AC) 63. (BC)

64. (AC) 65. (BC) 66. (ABCD) 67. (ABCD) 68. (ABCD) 69. (ACD) 70. (AB)

71. (CD) 72. (AC) 73. (BC) 74. (AC) 75. (AD) 76. (C) 77. (BD)

78. (ABC) 79. (ABD) 80. (ABD) 81. (ABCD) 82. (AD) 83. (ACD) 84. (CD)

85. (BD) 86. (ABCD) 87. (BD) 88. (BC) 89. (AB) 90. (ABC) 91. (ABCD)

92. (C) 93. (D) 94. (B) 95. (B) 96. (D) 97. (B) 98. (C)

99. (D) 100. (D) 101. (A) 102. (A) 103. (B) 104. (D) 105. (D)

106. (B) 107. (B) 108. (A) 109. (C) 110. (B) 111. (D) 112. (B)

113. (C) 114. (A) - (r), (B) - s, (C) - q, (D) - p 115. (A) - (r), (B) - (t), (C) - (q), (D) - (p)

116. (A) - (p,r), (B) - (p,s,t), (C) - (p,r), (D) - (q) 117. (A) -p,q,s,t ; (B) - p; (C) - s; (D) - p,s,t

118. (B) 119. 0 120. 4 121. 0 122. 5 123. 3 124. 2

125. 7 126. 6 127. 8 128. 4 129. 3 130. 1 131. 1

132. 2 133. 3 134. 1 135. 0 136. 1 137. 0 138. 7

139. 5 140. 2 141. 2 142. 9 143. 0 144. 4 145. 3

146. 5 147. 7 148. 5 149. 3 150. 1 151. 1 152. 6

153. 4 154. 1 155. 0 156. 8 157. 4 158. 4 159. 3

160. 2 161. 7 162. 3 163. 12 164. 00 165. 33 166. 16

167. 99 168. 47 169. 11 170. 13 171. 29 172. 02 173. 72

174. 05 175. 02 176. 05 177. 09 178. 24 179. 16 180. 43

RESONANCE Page - 21
MATHEMATICS

CRITICAL QUESTION BANK

SOLUTIONS
1. Method – I 5. Let 4m3 – 3am2 – 8a2m + 8 = 0 has three roots m1, m2, m3
f '(x) = 2sin(x + ) cos(x + ) + 2sin(x + ) cos(x + ) – 2cos
( – ) cos(x + ) sin(x + ) – 2cos( – ) sin(x + ) cos  m1m2m3 = – 2
(x + ) Given m1m2 = – 1
= sin(2x + 2) + sin(2x + 2) – 2cos( – ) sin(2x +  + )  m3 = 2  32 – 12a – 16a2 + 8 = 0
= 2sin(2x +  + ) cos( – ) – 2 cos( – ) sin(2x +  + )
=0
 f(x) is a constant function 12 3
Method – II  16a2 + 12a – 24 = 0  Sum of values of a = – =–
f(x) = 1 – cos2(x + ) + sin2 (x + ) – 2cos( – ) sin(x + ) 16 4
sin(x + )
= 1 – cos(2x +  + ) cos( – ) – 2cos( – ) sin(x + ) 6. P  (a  d)  pd
sin(x + )
= 1 – cos( – ) [cos(2x +  + ) – cos(2x +  + ) + Q  (a  d)  qd
cos( – )]
= sin2 ( – ) R  (a  d)  rd
a constant function
option A is correct
2. Product of roots For option B
c (a  b ) 2ac Tp  q r  (a  d)  (p  q  r) d
 =1  b=
a(b  c ) (a  c )
P  Q  R  3(a  d)  (p  q  r)d
b 2c 2 2  a  d need not to be zero
 = = – (+) =
a a c a 1
  1 1  Tp  q r  P  Q  R
c 
pqr
For option C
2 If Common diff. is  ve . then
 = – 1 – 
 pd  qd  qr  PQR
For option D not necessarily true. if first term is not an integer.
 1 2   a b   a  2c b  2d 
3. AB =     =   7. Let y = x2 – 4x + 5
 3 4   c d   3a  4 c 3b  4d 
y = (x – 2)2 + 1
 sin–1 y, cos–1 y defined only at y = 1
a b 1 2 at which x = 2
BA =    
c d 3 4 So given equation


 a  3b 2a  4b  4 + 2a + =0
=   2
 c  3d 2c  4d 
if AB = BA, then a + 2c = a + 3b 8
a
 2c = 3b  b0 4
b + 2d = 2a + 4b  2a – 2d = – 3b
8.   log 1015 = 1  log10 3  log10 2
9
 b = 4log10 2
3a  3d 2 
=
3b  c 3 = –1 4log10 3 = 4    4
3b  b
2 we required those elements of A which can be written in terms
4. We know lim x x  1 of log10 2 & log10 3 .
x 0
x i.e. number of integers which are divisible 2 or 3, or 5 only.
lim (x  1)  0
x  0 required numbers = 24
x
e( x 1)
 xx 9. Roots are 2k, – 3
lim 2 – 6 < 2k < 10 k  (–3, 5)
x 0 x
 (x²)  1 largest value of k is not defined.
 x (x x  1)² (x x  1)³  ( 2x  1)
1  (x  1)    ...   1  (x x  1)
2! 3! 10. 0
 lim   (2x  3 x 2  x )
3
x 2 2
x 0
 (x )  1
 1  1 
(x x  1)² (x x  1)³  x  (– , –1)  , 0    ,   
  ......
1 1 1
 2  2 
 lim 2! 3!  . 
2 2 for log2 x, x > 0
x 0 x x
 (x )  1  (x )  1 2 4 8
for sin 1(log2 x ) , 0  log2 x  1
 x  [1, 2] ........(2)

RESONANCE Page - 22
MATHEMATICS
So by (1) and (2) we get
x  [1, 2] 1
dx = I1 – I2
1 3  (sin x) 2010
11. R2 – SG2 = (a2 + b2 + c2)  (abc)2/3 .........(1)
9 9 Applying by parts on I1, we get

abc tan x tan x cos x


abc = 4R = 4Rrs = 4Rr    2Rr(abc)1/3.3  2010  (sin x ) dx
 2  (sin x )2010 2011

(abc)2/3  6Rr
tan x dx
1 =
2010
 2010  (sin x ) 2010
2
R – SG  2
6Rr (sin x )
3
SG2  R2 – 2Rr tan x P( x )
SG2  SI2  I1 – I2 = = + c  P(x) = tanx
SG  SI (sin x )2010 (sin x )2010
12. Image of the centre C2 (1, –3) in the line 3x + 4y –16 = 0 is P 
(7, 5).
Now for C1C2 + C2C3 + C3C1 to be minimum C1, C3 and P should
 P(x) = tan = 3
3
be on same line so C3 = (0, 4)
C3 = (0, 4) 
15. c  x ( i  j )  y( j  k )

c.( i  j )
   (i  j ).( j  k)
   

c 2 2. 2.

2x  y 1

2 2 2
x  (x  y)  y 2
Distance between C3 and C1
2(4x 2  y 2  4xy)  2x 2  2y 2  2xy
 50  5 2
radius of C1 = 3 2 6x 2  6xy  0
so radius of C3 = 2 2 x = 0 or x  y
Equation of C3 (x – 0)² + (y – 4)² = 8 
x² + y²– 8y + 8 = 0, a = 0, b = –8, c = 8
c  x (i  k)

13. For 0 < n < 1, sin x <sinn x and for n > 1, sin x > sinn x
Now, for 0 < n <1, 1  
c  (i  k)
n
2(sin x  sin x)  (sin x  sin x) 1 n 2
ƒ(x)  
2(sin x  sinn x)  (sin x  sinn x) 3 c.
 j  0
and for ,
16. (A + B)2 = (A + B). (A + B)
2(sin x  sinn x)  (sin x  sinn x)
ƒ(x)  3 = A 2 + AB + BA + B 2 .........(1)
2(sin x  sinn x)  (sin x  sinn x) Since B = – A –1 BA
 AB = A (–A –1 BA)  A(BC) = (AB) C
For n  1,g(x)  3,x  (0, ) = –(AA –1) BA
 AB = – BA
 put in (1), we get
 g (x) is continuous and differentiable at x  , for
2 (A + B)2 = A 2 – BA + BA + B 2
(A + B)2 = A 2 + B 2
0  n  1,
 2 
 1        3  1   3
 
  3   0, x   0, 2    2 ,   17.  = sin–1 
8
 + cos–1  2  + sec–1 ( 2)
g(x)         



 
 
3, x
 2  3  1  3
   

 = sin–1  2 2 
 
+ cos–1  2 
 
+ sec–1  2
 g (x) is not continuous at x  , Hence, g (x) is also not
2 
 
= + +
 12 6 4
differentiable at x 
2
  2  3  
= = .
12 2
sec 2 x  2010 2
14. dx =  sec x.(sin x ) 2010 dx
 sin 2010
x
– 2010
18. f is continuity iff x = cos x
which is possible only for one 'x'

RESONANCE Page - 23
MATHEMATICS
1
1  1   1
19. Since f(x) = 0 –
1  x3  ta n  2   4  co t 3 ............................(2)
 
2 2 2 From 1
 x2  x2
 xf(x)
 
dx =  f(x).  –  f (x) dx 13 1 1
 2 0 2   cot 3
0 0
24 6
2
1 x2 From 2
= 2f(2) +  dx  f(2) = 0
2 0 1  x3 
3
 cot 1 3
2 4
1 3x 2 2
dx = 1 ×  1  x3
1/ 2 
=0+
6
 1 x 3 6 
  
0   
3 13 

1
 1  cot 13
0
4 24  6 
1 2
= [3 – 1] =
3 3 5 5
  cot 1 3
20. Clearly %P  (1,2) 24 6
Equation of tangent at P is 4x (1) + y(2) = 8
 p  5, q  24, r  5, s  6
P ij Li'kZjs
[kk d k lehd j.k 4x (1) + y(2) = 8
 A  (2,0) and B = (0, 4) dy
24.  y. =1
 A  (2,0) vkS
j B = (0, 4) dx
Similarly normal at P is 2x – 4y + 6 = 0
blh izd kj P ij vfHky Ec 2x – 4y + 6 = 0   ydy   dx
y2
 =±x+c
2
 Q(0, 1) lies on curve 
2
y 1
=±x+
2 2 2
 y = 2x + 1 and y2 = – 2x + 1

3  1  1
 y2 = 2  x  and y2 = – 2  x –   Required area = 4

 A (–3, 0) and B  0,   A (–3, 0) rFkk 2  2
 2   
2 –1 1/ 2
3/2
 3 =4× 1– 2x  
B  0,  3 2
×
0
 2
–4 4
0 4 1 = (0 – 1) =
3 3
1 1 2 1 5
 Area BPB = = and area APA
A = 5 25. f(f(x)) = f(x) + f2(x) + f4(x) + f8(x) + .....
2 0 3/2 1 4
Coefficient of x10 in f(x) = 0
 Ratio = 4 Coefficient of x10 in f2(x) = 2
Coefficient of x10 in f4(x) = 4C2 + 4C1 = 10
Coefficient of x10 in f8(x) = 8C2 = 28
 x x  x / 2
 cos  sin e dx 2008
 2 2 
21. I=  x
26. Let S =  r( r   2009–r )
2 cos 2 r 1
S = 2009 (1 + 2 +.......2008)
2 S = – 2009
Put –x/2 = t
(cos x)
I=– e t (sec t  sec t tan t ) dt 27. lim 2 must be '0’
 x

 
= –et sec t + C
2
x – 2
 
x iff > 2
= –e–x/2 sec +C
2 28. Let variable line be x + my + n = 0

e 1 x – 2 e –1 – 2 1 3ax1 3amy1


22. L = lim– = =2–  n
x 0 1  x –1 e  P1 =
a  b  c a bc
 2  m2
23. tan1 1  tan1 2  tan1 3   3bx 2 3bmy 2
 n
1  a  b  c a  b  c
 tan 2   cot 1 3 ..............................(1)  P2 =
4  2  m2
 1  1 
tan1(1)  tan1    tan1   
2
  3 2

RESONANCE Page - 24
MATHEMATICS
3cx 3 3cmy 3  (n  1)a na 
 n 
 P3 =
abc abc 
2 2

 
 2  m2 = tan-1   (n  1)a   na  
Given P1 + P2 + P3 = 0 1   2 . 2  
    
3(ax1  bx 2  cx 3 ) 3m(ay1  by 2  cy 3 )
 + + 3n = 0  a  na 
(a  b  c ) (a  b  c ) = tan-1  (n  1)  – tan–1  
 2  2 
(ax1  bx 2  cx 3 ) m(ay1  by 2  cy 3 ) Put n = 1, 2, 3, .........., n we have
 + +n=0
(a  b  c ) (a  b  c )
(n  1)a a  a a
Hence line  x + my + n = 0 passes through incentre Sn = tan–1 –tan–1 ; S = – tan–1 = cot-1  
2 2 2 2 2
 ax 1  bx 2  cx 3 ay 1  by 2  cy 3 
 ,  of triangle ABC. 
 abc abc   a=2 S =
4
29. According to LMVT
35. 5x5 – 10x3 + x + 2y + 6 = 0 ......... (i)
f ( x ) – f (0 ) Differentiating w.r.t. x
= f (C1 )  1 dy
x 25x4 – 30x2 + 1 + 2 =0
dx
 f(x)  | x|
dy 1
= – (25x4 – 30x2 + 1)
f ( x ) – f (0) dx 2
again by LMVT = f (C2 )  | x |
x  dy  1 1
 
 dx (0 ,–3 ) = 2 ( 0 – 0 + 1) = 2
– –
 f(x)  x2
 equation of the normal at P(0,–3) is P(0,–3)
y + 3 = 2 (x–0)
30. f(x) = cos 8 {x} = cos (8 x – 8 [x]) = cos 8x. Its period is
 y = 2x – 3 .......... (ii)
1 solving equation (i) and (ii)
. 5x5 – 10x3 + x + 4x – 6 + 6 = 0
4 5x (x4–2x2+1) = 0
5x (x2–1)2 = 0
1 1 x=0,1,–1
Period of sin 2x cosec2x is  period of f(x) is
2 2 y=–3,–1,–5
 Normal at point P(0,–3) meets the curve again at two points
32. f(0) = k and xlim
0
f(x) = xlim
0
x2{e1/x} = 0 (1,–1) and (–1,–5) .
Now equation of tangent at point (1,–1) is
 for f(x) to be continuous f(0) = xlim
0
f(x)  k = 0  dy 

y + 1 =  dx 
 (x–1)
2 1/ h (1,–1)
 f (0+) = lim h {e } = 0  y + 1 = 2 ( x –1)

h 0 h  2x – y – 3 = 0 ........ (iii)
Equation of tangent at point (–1, –5) is
2 –1/ h
 f(x) is derivable at x = 0 and f (0–) = lim h {e }
=0
h 0 
–h  dy 
y+5=   (x+1)
due to {e1/x}, will be discontinuous at all value of x, where e1/x  dx  (–1,–5 )
becomes an integer.
 y + 5 = 2 (x + 1)  2x – y – 3 = 0 ........ (iv)
i.e. at x = log2e, log3e,.....etc.
33. f is even and g is odd 36. f(0) = 1
 gogog is odd gogog 2

 fogogog is even fogogog


and f(x + y + 1) =  f (x)  f (y) 
substituting x = 0, y = 0, we get f(1) = (1 + 1)2 = 4
 gofogofogogog is even. gofogofogogog
2
n(n  1)
substituting y = 0, we get f ( x  1)   
f (x)  1
34. nth term of 1 + 3 + 6 + 10 + ....... is n = f(0) = 1,f(1) = 22, f(2) = 32, f(3) = 42 and so on
2
hence f(x) = (1 + x)2
 1 n(n  1)a   4  n(n  1)a 2 
37. Composition of one - one function is one - one
T n = cot –1 2a   = cot –1  

 2   2a 
38.  AQ = a2 t 24  4a2 t 22 = at2 t 22  4 = t 2 t 22  4
 a 
  ( a = 1)
 2a   2 
–1
=tan 
4 n( n 1)a
–1
2  = tan  na (n  1)a  2  –2 
     1  
2 2 Since t2 = – t1 – = (– t1) +  
  t1  t1 

RESONANCE Page - 25
MATHEMATICS
 t1 < 0 (as shown is diagram x1 = 3(h – 2) and y1 = 3(k – 2)
now apply A.M.  G.M orthocentre (x1, y1) lies on x2 – y2 = 36
 9(h – 2)2 – 9(k – 2)2 = 36
(h – 2)2 – (k – 2)2 = 4
 locus of centroid G(h, k) is (x – 2)2 – (y – 2)2 = 4
 = 4

4 – x 2 2x3
44. Here f (x) =  ,
 1 x3
 –2 
(–t1 )     f(x) is decreasing in [2, 3) and increasing in [3, ).
  t1   2 Now, f(x) will have least value at x = 3 if
2 = lim f(x)  f(3) i.e. – 15 + n(b2 – 3b + 3)  – 15
x 3 –
 –2  i.e. n (b2 – 3b + 3) 0
 (–t1) +   2 2  t2 2 2  b2 – 3b + 3  1 i.e. b2 – 3b + 2  0
 t1  i.e. b (–, 1]  [2, ).

 (AQ)min = 2 2 84 = 4 6 45. Let cosec–1x = t

39. p = (x – 2) (x – 4) = (x2 – 6x + 8)



 t2 – 2t  (t – 2)
p(x) = (x3/3 – 3x2 + 8x) +  6
2  (t – 2)(t – /6)  0
40. y = f(x) = (3x + 2)2 – 1, – < x  –  t  /6 or t  2 (not possible)
3  t  /6
 0+ < t  /6 or –/2 t < 0–
 0+ < cosec–1x /6 or –/2 cosec–1x < 0–
 2 x <  or –  < x  –1
 x [2, )  (– , –1]
 a = –1, b = 2  a+b=1

It is clear from the graph that if x  (–, –2/3], then y  [–1,)  1  Z1  Z2  .........  Z7 
46. arg  
y 1  1  Z8  Z9  Z10  .......  Z14 
2
y + 1 = (3x + 2)  3x + 2 = ±
( x  –2/3,  negative sign)

3x + 2 = – y 1  1  Z1  Z 2  .........  Z7 
= arg  
–2 – y  1  1  Z1  Z2  ..........  Z7 
 x= = g(y) = f–1(y)
3
= 2 arg (1+Z1 + Z2 + .........+Z7)

–2 – x  1  Z 
 g(x) =
3  arg = 2 arg (Z) 
 Z 
9 2 42
41. cot 1  tan 1  tan 1  tan 1 4  tan 1 2 arg (Z3+Z4)
2 9 1  4.2
33 4 84
cot 1  tan 1  tan 1  tan 1 8  tan 1 4 z4 z3
4 33 1  4.8 z5
z6 z2
z1
129  8  16  8 z7
cot 1  tan1   tan1  tan1 16  tan1 8
8  129  1  16.8 1

Similarly Tn = tan 1 2n1  tan 1 2n on adding we get


= 2 arg (Z3 + Z4)
sum = tan 1 2n 1  tan 1 2
 =2x 7  14
as n   sum = – tan–1 2 = cot–1 2.

2 15 15
42. Let S = 2C0 + 3C1 + 4C2 + 5C3 +.......+ 99C97 + 100C98 Also arg (Z1 Z2 Z3.......Z7)
 S = 3C0 + 3C1 + 4C2 + 5C3 +.......+ 99C97 + 100C98
S = 4C1 + 4C2 + 5C3 + .......+ 99C97 + 100C98 2 4 6 14
S = 5C2 + 5C3 + .......+ 99C97 + 100C98 =    .......... 
S = 101C98
15 15 15 15
43. Orthocentre lies on the rectangular hyperbola and 2 56
= (1+ 2 +3 +.......+7) 
15 15
2(3 )  x1 2(3)  y1
 h= and k =
3 3

RESONANCE Page - 26
MATHEMATICS
1 b
y= x ..........(2)
47. x + x2  1 = y  y2  1 a
b
  y=– x .......(3)
1 a
take loge n ( x  x 2  1)  n
 
 2  M  [a(sec – tan), b(sec – tan) ]
 y  y 1 and N  [a(sec + tan), b(sec + tan) ]

 y2  1  y 
 ON = a 2  b 2 (sec + tan) = ae(sec + tan)
   OM + ON = ae(2sec) = 2ae(sec)
= n  2 2 
 y  1 y 
  a a
SP + SP = e(asec – ) + e(asec + ) = 2aesec
e e
x2  1 + x = y2  1 – y

x+y= x2  1 + y2  1 .....(1)
D
1 1 1
2 50. V = AD (
similarly y + y 1 = 3 2
x  x2  1

 y2  1 + y = x2  1 – x

x+y=– x2  1 – y2  1 .....(2) B
adding (1) & (2) x + y = 0 90° 30°
Image of xy = 1 in xy = 1 it self. A

48. |1+ Z1 + Z2 + Z3 + ..........Z7|


= | 1 + Z7| + |Z1 + Z6| + |Z2 + Z5| + |Z3 + Z4|

7 5 3 
= 2cos + 2cos + 2cos + 2cos
15 15 15 15
C
 4 3 4 
=2  2 cos 15 cos 15  2 cos 15 cos 15  AB. BC sin 30°)
   AD. AB. BC = 216
4  3  Now, AB + BC + AD  3 (AD. AB. BC) 1/3

= 4 cos
15 cos 15  cos 15   AB + BC + AD  18
 
and minimum value occurs when AB = BC = AD = 6

 2 4 8 sin(8 / 15)
= 8 cos cos cos =
Hence AC = AB2  BC 2  2AB  BC cos 30
15 15 15 8 sin(  / 15)
=3  6 2 
1 7
= sec 52. Since f(x) > 0
2 15  f (x) is always increasing.
g(x) = 2f (2x3 – 3x2) × (6x2 – 6x) + f (6x2 – 4x3 – 3) (12x – 12x2)
= 12(x2 – x) . (f (2x3 – 3x2) – f (6x2 – 4x3 – 3))
= 12x (x – 1) [f (2x3 – 3x2) – f (6x2 – 4x3 – 3)]
For increasing g(x) > 0
Case-I x < 0 or x > 1  f (2x3 – 3x2) > f (6x2 – 4x3 – 3)
 2x3 – 3x2 > 6x2 – 4x3 – 3 { f(x) is increasing}
49.
1  1 
 x   > 0  x > – 1  x   – , 0   (1, )

 (x – 1)2
 2 2  2 
Case-II If 0 < x < 1
x2 y2 f (2x3 – 3x2) < f (6x2 – 4x3 – 3)
– 1
a2 b2  1 1
(x – 1)2 x   < 0  x<– , so there
Tangent at P(asec, btan)  2 2
x y is no solution
sec   tan   1 ........(1)
a b  1 
Asmptotes are  Hence the values are x    ,0   1,  
 2 

RESONANCE Page - 27
MATHEMATICS
53. Clearly, the point lies on 7x – y = 5
2
lim f ( x  h )  f ( x ) ( f (h))2  f ( x ) – f ( x )
 = hlim
h0
h2 0
h2
= (f (0))2
 f (x) = (f (0))2  f (0) = (f (0))2
 f (0) = 0 or f (0) = 1
 f(x) = (f (0))2 x
Also, centre of the circle must lie on the bisectors of the lines x +
y + 13 = 0 and 7x – y – 5 = 0 given by Now (i) if f (0) = 0, then f(x) = 0 = x f (0)
(ii) if f (0) = 1, then f(x) = x = x f (0)
x  y  13 7x  y  5
  x  3y  35 and 3x  y  15  0 56. f(0) = r is odd. Let r = 2n + 1, n  I.
2 50 f(–1) = –1 + p – q + 2n + 1 = p – q + 2n is odd
Let (h,k) be the centre of the circle, then h -3k = 35 ..... (1)  exactly one of p, q is odd
f(1) = 1 + p + q + 2n + 1 = p + q + 2n + 2 is odd
and 3h + k +15 = 0
If possible suppose ,  I be zeros of f(x).
..... (2)
 x3 + px2 + qx + r = x3 – (+ + )x2 + (+ + )x – 
Clearly CA is perpendicular AQ  r = –   are odd integers
 p = – (+ + ) is odd
k2 and q = + + is also odd.
  7  1  h  7k  15  0 ..... (3)
h 1 It is a contradiction. Hence f(x) = 0
cannot have three integer roots.
On solving, we get centres as C1 (29, – 2) and C2  (– 6, 3)

 r21 = 800 and r21 = 50  Smaller circle has radius = b c


57. f(x) = x2 + x+
a a
50 f(–1) < 0 & f(1) < 0

Therefore area of quadrilateral ACBQ = 50  200 sq. 1 log x5


units = 100 sq. units. Hence (A) 58. 1 – log 3 2 ( x  1) 2 = 
31/ 2 x  3

2  
54. z = x + iy
(x – 2)² = (x – 7)² + ( y + 2)² x5
(x – 2)² - (x – 7)² = ( y + 2)²
2 2
 1– log3 |x + 1| = log3  
2 2 x3

 3  x5
 log3   = log  
 | x  1|  3
x3

3 x5
 =
| x  1| x  3
Case- I x+1>0  x>–1
 3(x + 3) = (x + 1)(x + 5)
( y + 2)² = 5(2x – 9)  x2 + 3x – 4 = 0
 x = – 4 or x = 1
 9 x = – 4 rejected ( x > – 1)
(y + 2)² = 10 x    x=1
 2
Case - II x+1<0  x<–1
 3(x + 3) = – (x + 1)(x + 5)
5  x2 + 9x + 14 = 0
Y²  4aX  a 
2  x = – 2 or x = –7
PQ |min  L(LR)  10  Set of value of x = {–7, –2, 1}

m
lim  lim x f ( x )  h( x )  3 
55. (x2 + y) = (f(x))2 + f(y)  
f(0) = (f(0))2 + f(0)  f(0) = 0 59. lim g(x) = = m
lim
x 1 x 1 m
for y = 0 : f(x2) = (f(x))2 ........(i) m 2x  4 x  1 
for y = – x2 : 0 = f(0) = (f(x))2 + f(–x2)
 (f(x))2 = – f(–x2) ........(ii)  x m f ( x )  h( x )  3 
from (i) and (ii) lim  
f(–x2) = – f(x2) x 1  2x m  4 x  1 
 
 f(–x) = –f(x) ........(iii)
thus f(x) is an odd function
if f(x) is even also, then f(–x) = f(x) ........(iv)  h( x )  3 
 f(x) = 0 for all x {by (iii) and (iv)}  f ( x ) 
x m  f (1)

since f(x) is continuous at x = 0, = mlim lim   =
 2  4x  1 
  x 1 2
 lim f(h) = 0  x m 

h0
f(1) = 2e3
 lim
2 f(x + h2) = lim
2 {(f(h))2 + f(x)} = f(x)
h 0 h 0
lim h(1)  3
 it is continuous everywhere Similarly g(x) =
x 1– 5
since lim f (0  h)  f (0) = hlim
f (h )
exists
 h(1) = 5e3 – 3
h0 h 0 h

RESONANCE Page - 28
MATHEMATICS
60. The tangent 3x + 4y – 25 = 0 is tangent at vertex and axis is 4x  for rational terms r = 0, 2, 4, 6, ....... 14
– 3y = 0 66. Solution set = 
So, PS = a = 5 67. A & B are obvious
Latus rectum = AB = 20
 1  1  1 
f   -1 = i.e., f   -1isreciprocalof (f(x)-1)
61. Let 768  32 cos    x   (f(x)-1)  x 
16 3  32 cos   /2 cot-1 | x|  1  tan-1 | x|
Now,forf  x = -1
,f(x)-1= -1 ,f   -1= -1
3  tan | x| tan | x|  x  cot | x|
cos   
2 6 2 1-k  n| x|  1  1+k  n| x|
Alsoforf(x) = ,f(x)-1= f   -1=
So 4  8  32  32 cos
 1+k  n| x| 1+k n| x|  x  1-k  n| x|
6
 68. t1t2 = 2 and t1 + t2 + t3 = 0 and a = 2
 4  8  8 cos
12
 11
 4  4sin  4  4 cos
24 24 P(t1 )
T
 11  11 11
 4 1  cos   4.2 cos ²  2 2 cos Q(t2 )
 24  48 48
So a  2, b  48
b
  24
a

R(t3 )
62. f(x) = ex (x2 + bx + c) > 0 iff D = b2 – 4c < 0
g(x)= ex (x2 + (b + 2)x + (b + c)) > 0 iff D=b2 – 4c + 4=D + 4 < 0
 Let circumcentre of TPQ be O(h,k)
2
 
  O(h, k) will be mid point of RT
 1
63.  2 cos1 x   a  2 cos1 x
 a2  0



  2  at1t 2  at 32
h=  h = 2 + t32 ........ (1)

2
let cos1 x =t  t2
2 a(t1  t 2 )  2at 3
and k =  k = t3 .........
1 2
t2   a   t  a2  0

Now equation has one root 2 or (2)
 2   h = 2 + k2
greater than 2 and other root less than 2  locus of O(h, k) is
 O(h, k)
 ƒ(2)  0 y2 = x – 2
y2 = 1(x – 2)
 1
 4   a  2  2  a  0
2

4  2a  1  a2  0
a2  2a  3  0
(a  3)(a  1)  0  a  3 or a 1 69.

64. f is discontinuous at points outside its domain.

65.  Tr+1 15Cr(x4)(15–r)(x–3)r = 15Crx60–7r


(A) for the term independent of x, 60 – 7r = 0  r is not an
integer 70. a0z4 + a1z3 + a2z2 + a3z + a4 = 0
 there is no term independent of x. Taking conjugate on both sides.
(B)  n = 15 is odd 4 3 2
a0 ( z ) + a1 ( z ) + a2 ( z ) + a3 z + a4 = 0
n–1 n 1
 nCr will be maximum if r = or r =  z1, z 2 , z 3 , z 4 are the roots of the equation if z1 is real,
2 2
i.e. r = 7 or r = 8 then z1 is also real and if z1 is non real, then z1 is also root
 binomial coefficient of 8th and 9th terms will be greatest because imaginary roots occur in conjugate pair.
(C) for the cofficient of x32; 60 – 7r = 32  r = 4
 cofficient of x32 is = 15C4 sin(e x–3 – 1)
for the coeff. of x–17; 60 – 7r = – 17; r = 11 71. lim
X 3 n(x – 2)
 coeff. of x–17 is = 15C11 = 15C4
 (C) is correct put x = 3 + h
60–7r
(D) If x = 2 , Tr+1 = 15Cr 2 2

RESONANCE Page - 29
MATHEMATICS
h h h x
sin(e – 1) sin(e – 1).(e  1) h 2 1 1
lim = lim  76. f (x)   is one-one function.
h0 n(1  h) h  0 h
(e – 1)h n(1  h) 4x  1 2x  1
=1×1×1 =1
77. Let S  x2 + y2 + 2gx + 2fy + c = 0

        it cuts x2 + y2 = 4 orthogonally
 ab . bc 
     b.c a.b  a.c  c=4
       Moreover – 2g + 2f + 9 = 0
 bc . ca 
     a.c b.c  a.b ( (– g, – f) satisfy the given equation)
72.  S  x2 + y2 + 2gx + 2fy + 4 = 0
      
ca . a  b  a.b a.c  b.c  x2 + y2 + (2f + 9)x + 2fy + 4 = 0
       (x2 + y2 + 9x + 4) + 2f (x + y) = 0
It is of the form S + P = 0 and hence passes through the
Given that intersection of S = 0 and P = 0 which when solved give (– 1/2,
1/2), (– 4, 4).
            1
a  b  c  3  a  b  c . a  b  c  3  a.b  b.c  c.a  0
   78. Replace x by 2, 2f(2) + 2f   -2f(1) =4
         2
  a.b . b.c  b.c c.a  c.a a.b  a.b  b.c  c.a
          1
       f(2) + f   =2 + f(1)-----(1)
   a.b . b.c  b.c c.a  c.a a.b
          2
Replace x by 1, f(1) = -1 ..............(2)
   0[since , x  y  z  0, xy  yz  zx  0]
  
max  0only when a.b  b.c  a.c  0 1 1 1 5
Replace x by , 2f   + f(2) + 2 = ..............(3)
      2 2 2 2
 a  b, b  cand c  a
         1
2a  3b  4c . a  b  5b  c  6c  a
  Solve (1) and (3) => f  = 0 ; f(2) = 1
         2
 10a.(b  c)  18b.(c  a)  4c.(a  b)  32

73. As the point (, ), lies on both f(x) and g(x) , the point () 79. L et 7 6 8  3 2 co s 
will also lie on both the curves and as the functions are
continuous they must cross (meet on) the line y = x in between. 1 6 3  3 2 co s 
f must be on decreasing path, for all these to happen.
3 
co s     
74. f (x)= ab cos x – b 1 a 2 sin x = b cos (x + ) 2 6

  
So 4 8 3 2  3 2 co s  4 8  8 co s
Where  = cos–1a. So f(x) = 0  x =(2m +1)  6 12
2
 1 1
   4  4 sin  4  4 co s
Also f(x) = –b sin (x+). Thus f       < 0 and f
24 24
 2   1 1  1 1 1 1
 4  1  co s   4 .2 co s ²  2 2 co s
 24  48 48
 3 
     > 0 S o a  2, b  4 8
 2 
 (b – a )  46
 3
Hence f has maximum at –  and minimum at .
2 2
80. Equation of the required plane is

More over, max f = ab sin (/2 – ) +b 1 a 2 cos ( 2
)+ _ _ _ _ _ _ _
(r.n1  q1 )   ( r.n2  q2 )  0  r.(n1   n 2 )  (q1  q2 )
c = a2b + b (1– a2) + c = b + c
_ _
and min f = – b + c.
Now , perpendicular to n3 n 4
and min f = – b + c. _ _ _ _
(n1   n 2 ).(n 3  n 4 )  0
75. lim (log 3 (ax 2  3x  1)) must be equal to 1
_ _ _
x 2
[n1 n 3 n 4 ]
So 4a + 7 = 3  a = – 1  _ _ _

 log 3 (  x 2  3x 1) 1   ( x 1) (3  2x ) 


[n 2 n 3 n 4 ]
lim   lim  
x2 log 3 ( x 1)  x  2  (  x 2  3x 1)  So independent on q3 and q4 only.
Also le  
e  
 e 1/ 3

RESONANCE Page - 30
MATHEMATICS
81.
x  1 y  0 z  1  (1  0  2  4)
   2 , or
1 2 2 1  ( 2)2  22
L et C  ( 1  1, 2  1  2 , 3  1  3 ) a n d
D  (  1, 2  2, 3  3). 2 2 5
2 2 2 Q( x, y, z )   , , .
CD  14 3 3 3 
 ( 1   2 )²  4 ( 1   2 )²  9 ( 1   2 )²  1 4 84. (A) Consider g(x) = sin–1 f(x) – x
  2  1  1   2  1  1

L et G  ( ,  ,  ) since g(0) = 0, g   =0
4  5  21 2
4  10  41  
4  9  61  there is at least one value of    0,  such that g ()
 2
4 x -5 4 y -1 0 4 z -9
L o c u s o f G is = =
2 4 6 f (  )
5 5 9 = –1 = 0
2x- y- z- 1 – ( f (  ))2
2 = 2 = 4
1 1 3
2 i.e. f () = 1  ( f ( ))2 for atleast one value of  but may

 
not be for all    0,   false
3x  6y  2
82. =1
k
2x
........ (1) (B) Consider g(x) = f(x) –
2x2 + 2xy + 3y2 – 1 = 0 ........ (2) 
Homogenizing (2) with the help of (1), we get
2

 3x  6y  since g(0) = 0, g   =0
2x2 + 2xy + 3y2 –   =0 2
 k 
 k2(2x2 + 2xy + 3y2) – (3x + 6y)2 = 0 ...... (3)  
 there is at least one value of    0,  such that g ()
 circle described on AB as Diameter passes through (0, 0)  2
 AB will subtend right angle at (0, 0)
 coefficient of x2 + coefficient of y2 = 0 2
 (2k2 – 9) + (3k2 – 36) = 0  k2 = 9  k = ± 3 = f () – =0

2
i.e. f () = for atleast one value of  but may not be for all

 
  0, 
 2
83.  false

2x
(C) Consider g(x) = (f(x))2 –


The rod sweeps out the figure which is a cone. since g(0) = 0, g   =0
2
The distance of point (A(1, 0, – 1) from the plane
 
1 1 2  4 |  there is at least one value of    0,  such that g ()
is  1 unit.  2
9
The slant height l of the cone is 2 units. 2
= 2f() f () – =0
Then the radius of the base of the cone is 

l2  1  4  1  3 1
 f() f () =  true


Hence, the volume of the cone is ( 3 ) 2 .1  
3 4x 2
(D) Consider g(x) = f(x) –
cubic units. 2
Area of the circle on the plane which the rod

traces is 3. since g(0) = 0, g   =0
Also, the centre of the circle is Q(x, y, z). Then 2

RESONANCE Page - 31
MATHEMATICS
  6 1
 there is at least one value of    0,  such 89. (A)  f(x) = 1 – x6/5  f (x) = – exist x  (–1, 1)
 2 5 x5
and  f(1) = 0 = f(–1)  Rolle’s theorem is applicable
8
that g () = f () – =0 x x 2 
2 2
xe t dt x  e t dt 
(B)  xlim
 = xlim
  
8 0 0 0 0 
f () = true

2

1  x – ex 1 x – e x

x
x2 2
x(e )  e t dt  0  
= xlim
0 0 0
 
85.
1– e x
2 2 2
x(e x .2x)  e x  e x 0  1 1
= xlim = = =–2
 third side will be parallel to bisectors of two given line
0
–e x –1
 
7x – y  3 (–x – y  3) (C)  | a + b | = 1 + 1 + 2cos= 2(1 + cos ) = 4 cos2/2
 Bisectors are =±
5 2 2
 2
 2
 Bisectors are 3x + y – 1 = 0 and x – 3y + 9 = 0 and | a – b | = 1 + 1 – 2cos = 2(1 – cos) = 4 sin /2
Now required third side will be parallel to these bisectors
 3x + y + 7 = 0 or x – 3y – 31 = 0 1 1
   +  
86. f(x) = cos–1 (cos 2x), g(x) = |cos x| | a  b |2 | a – b |2
f(x), and g(x) both are even and periodic so max {f(x), g(x)}
and min{f(x), g(x)} will also be periodic and
even. 1  sec 2   cos ec 2  
= 
4  2 2 

1  2  2 
=  1  tan 2  1  cot 2 
4  

1  2  2 
=  2  tan 2  cot 2   A.M.  G.M
4  
but max {f(x), g(x)} will be non-differentiable when f(x) = g(x)
no of points where f(x) = g(x) are four in [0, 2]
87. f(x) = 0  2cosx + 2cos2x = 0
 2
 tan2 cot 21
2 2

 x= ,  are critical points
3 (D)  M(1, 2) lies on the director circle of 7x2 – 12y2 = 84

    3 
f(x) is increasing in 0, 3  and decreasing in 3 , 2  90. m3 + (2a + 5)m2 – 6m – 2a = 0
   

m1 + m2 + m3 = – (2a + 5)
 3 3  3  m1 m2 + m2 m3 + m3 m1= – 6
Hence f(0) = 0 , f   = , f() = 0 , f   =–2 m1 m2 m3 = 2a
3 2  2  For (A)
3
3 3
Least value = – 2, Greatest value =
2 a+ mi 1
i =–1

88. (A) If function is one- one & onto then a = b since every
 a – 2a – 5 = – 1  a = – 4  m1 m2 m3 = –8
element of set B should have exactly one pre-image
 m1 = 1, m2 = –2, m3 = 4  m1 m2 m3 = – 8
in A.
for (B)
(B) For one - one, into function, every element of set B should
a – 2a – 5 = – 5  a = 0  m1 m2 m3 = 0
have either one pre-image or no pre-image in
 m1 = 1, m2 = 0, m3 = –6
set A.  no. of elements in set B > no. of elements in
3
A.  b > a
(C) For many - one, onto function, every element of set B should
have one or more than one pre-images in
 a+ m
i 1
i =0+0=0 (C is correct)

A.  n for (D)
a + 2a = 32 not possible
(B) < n (A)  b < a or a>b
Hence (D) is incorrect.
(D) For many - one, into function aR & b R .
91. fof(g(x)) = 1 + sin(fog(x))  f(1 – x) = 1 + sin(1 – x)
 f(x) = 1 + sinx

RESONANCE Page - 32
MATHEMATICS
92. S1 : a, b, c must belongs to R.
dy
S3 : z = a + ib = a – ib = tan(sec–1 2)=1
 dx x 0
S4 : a, b  R
P(0) = 1  b = 1
93. S1 : D = 0  roots are real and equal if a, b, c  R.
S2 : Roots are integers
ax3 ax 2
 P(x) = – +1
3 2
a1 b1 c1
S3 :  
a 2 b 2 c 2 is true iff roots are both common ax 4 ax
3
 P(x) = – +x+c
S4 : If Numerator and Denominator have common factor then Rf 12 6
can be equal to Rg.
 1 1
P(–1) = 1  a  12  6  – 1 + c = 1
F2  
P( ) F
1

S S a
G  + c = 2 .........(1)
94. 4
and P(1) = 1  a = 12c .........(2)
solving (1) and (2), we get

Let I(h, k) and G(ae2cos, 0) 1


c= and a = 6
2

e(a cos )  ae2 cos  ae cos (1  e) x4 1


h= = = aecos  P(x) = – x3 + x +
1 e 1 e 2 2

e(b sin )  0 be 1
and k = = sin P(0) =
1 e 1 e 2
P(x) = 0  2x3 – 3x2 + 1 = 0 (2x + 1)(x – 1)2 = 0
be 1
h = aecos ; k = sin  a = 6, b = 3 3 and e = 99. to 100.
1 e 2
x y
 h = 3cos, k = 3 sin  cos–1 + cos–1 =
2 3
x2 y2
 locus of incentre I(h, k) is
9

3
=1 xy x2 y2
 cos = – 1 1
6 4 9
3 2
and its eccentricity = 1– =
9 3 x2 y2  xy 
 cos  
1 1 = 
we know that the straight lines joining SF1 and SF2 bisects the 4 9  6 
normal PG.
 Required point of intersection is midpoint of PG. 2   2  2
1  x  1  y 
  xy 
=   cos  
  9 3   4   9  6
 = P  3, 2  and G(ae2cos,0)  G  4 , 0 
P
  
3    
x2 y2 x2y2 x2y2 xy cos 
 15 9   1– – + = + cos2 –
4 9 36 36 3
 Required point =  8 , 4
 
x2 y2 xy cos
95 to 96  1 – cos2 = + –
f(1.1) = f(1.2) 4 9 3
 f is may one  9x2 + 4y2 – 12 xycos = 36 sin2
log{x} [x] < 2  [x] > {x}2
 N = 36
 [x] > (x – [x])2
and (cos–1 x)2 – (sin–1 x)2 > 0
 x2 – 2[x] x + [x]2 – [x] < 0
x [–1, 0) x2 + 2x + 2 < 0, 
not possible  cos 1 x  sin1 x > 0
 
x [0, 1)  x2 < 0, not possible 2
x (1, 2)  0 < 2  infinitely may solution.  1 
 x  1,    [p, q)
 2
97 to 98.
two points of inflection occurs at x = 1 and x = 0
 P(1) = P(0) = 0 1
 P(x) = ax(x –1)  p=–1, q=
2
3 2
ax ax
 P(x) = – +b
3 2

RESONANCE Page - 33
MATHEMATICS
 1     
99.  N–6 = 36 –6=0   1,  3sin–1(sinM) = 3sin–1  sin 12  =
 2   4
100. sec–1x is not defined at x = 0  x2 – (tan(3sin–1(sinM))) x + a4 = 0
 x2 – (tan/4) x + 3 = 0
101. to 102. x2 – x + 3 = 0
  +  = 1,  = 3
 –26 –10   () – (+) = 3 – 1 = 2
P  4, ,
 3 3  105. to 106.
 
 AB = ˆi – ˆj  3kˆ , AC  – ˆi – ˆj  2kˆ and BC  –2iˆ – kˆ 2
Roots are 1, –
AC  BC a
 Fourth vertex D is (4, 2, 0) 2
  – <0  a>0
AB × AC = ˆi – 5ˆj – 2kˆ a
2
 Equation of base is x – 5y – 2z + 6 = 0  a + 2a + 5 (5, )
Let E(x,y,z) be the foot of the perpendicular drawn from P to the
base 2  1
– < – 4 a   0, 
a  2
x – 4 y  26 / 3 z  10 / 3
 = = = r (let)
1 –5 –2 107. to 108
Given, (a + b)2 x – (ab + bc + ca + 1) y + 2 = 0
26 10
 (4 + r, – 5r – , –2r – ) Now, (a + b)2 – (ab + bc + ca) + 1  0 for all a,b  R
3 3  a2 + b2 + ab – bc – ca + 1  0
It lies on the base  a2 + (b – c) a + (b2 – bc + 1)  0 a R
 (b – c)2 – 4(b2 – bc + 1)  0  3b2 – 2bc + (4 – c2)  0 
 26   10  b R
 4 + r – 5  –5r –  – 2– – 2r  + 6 = 0  4c2 – 12(4 – c2)  0  c2 – 12 + 3c2  0  c2  3
 3   3   M=3
 r=–2 Also, lx + my + n = 0 (l, m, n are 1st, 3rd and 7th terms of an
arithmetic progression)
 4 2  lx + (l + 2d)y + (l + 6d) = 0
 E  2, ,  +m+n=4 ('d' be their common difference ) 'd'
3 3 

  l (x + y + 1) + d(2y + 6) = 0  x + y + 1 = 0, 2y + 6 = 0
  x = 2, y = – 3
EP = 2iˆ – 10ˆj – 4kˆ Hence  = 2,  = –3

height = EP = 120 x2 + y2 = 4 ; x2 + y2 – 24x – 10y + 2 = 0
c1(0, 0) ; c2 = (12, 5)
1    1
 Volume = |
3 AC BC EP
× || |=
3 25  1  4 120 r1 = 2 ; r2 = 169 –  2

60 169 – 2 > 0   – 48 , 48 
= = 20 cubic unit Since  is integer
3
 = 0, ±1, ±2,....,2 ±6
103. to 104  Number of possible integral values of  is 13 Ans.
x2 + y2 = M + 2 = 3 + 2 = 5
  tangent at (1, – 2)
 2( 3 – 1) 
  2 3 3
f(x) = tan–1  2 3  x + 2  2 3  x2 + 2 + 2
x  2 2 x x
 x 

 2( 3  1)

 2( 3 – 1) 
 fmax = tan–1   =  = M which occurs at x2 = x – 2y – 5 = 0 .......(1)
 2( 3  1)  12 This line touches the circle x2 + y2 – 8x + 6y + 20 = 0

3 2x  y  5 
i.e. at x = 31/4 = a Solving  x  3, – 1
x 2 x – 2y  5
and fmin = 0 = m at x = 0 Hence point of contact (3, – 1) Ans.
cos–1x + cos–1y = 109 to 110
( – ) = (( + ) – ( + ))
( + )2 – 4 = [( + )+ ( + )] 2 – 4( + ) ( + )
 –1  7  –1  3    7 3  (–b1)2 – 4c 1 = (–b2)2 – 4c 2
3  tan  tan 24   tan  tan 8    3  24  8   2 D1 = D2 ..........(i)
      
 x=y=–1  x+y=–2 D1 1
minimum of y = f(x) is – =–
4 4
 D1 = 1  D2 = 1

RESONANCE Page - 34
MATHEMATICS
D2 1 13 14 0  1 2 0   41 40 0
 minimum of y = g(x) is – =–
4 4     
14 13 0 2 1 0  40 41 0

P4 = P3P =  =
b2 7  0 0 1 0 0 1  0 0 1
minimum of y = g(x) occur at x = – =
2 2
 b2 = –7  41 40 0  1 2 0 
b22 – 4c2 = D2    
40 41 0 2 1 0 
Also P5 = P4P = 
48  0
49 – 4c 2 = 1  = c2 0 1 0 0 1
4
 c 2 = 12
121 122 0 
111 to 113  
122 121 0 
= 
As det. (P – nI) = 0  0 0 1
1– n 2 0  Tr.(P) = 3, Tr. (P3) = 27 = 33
Tr.(P5) = 243 = 35  Tr. (P2011) = 32011
 2 1– n 0 =0
Also Tr.(P2) = 11 = 32 + 2
0 0 1– n Tr.(P4) = 34 + 2
Tr.(P2012) = 32012 + 2 Ans.
 (1 – n)(n – 3)(n + 1) = 0 n = –1,1,3
114. (A) logsinx (log3 (log0.2 x)) < 0 = logsinx1
–1 0 0  log3 (log0.2x) > 1 log0.2x > 3 = log0.2(0.2)3
  1
0 1 0
So N =   0 < x < (0.2)3  0<x<
 0 0 3 125
 det.(N) = – 3  det.(adj N) = (detN)2 = (–3)2 = 9 Ans. (e x  1)(2x  3)( x 2  x  2)
T (B) 0
112.Given PQPT = N  P

–1
PQ P (P T ) –1 = P–1N (PT)–1 (sin x  2) x( x  1)

 Q = P–1N(P–1)T [As (PT)–1] = (P–1)T]
 Q

–1 2 –1 2  1  ( e x  1)( x  3 / 2) 3


 0  0  0 0   0  x < – 1 or x 
 3 3 x( x  1) 2

–1 0 0  3 3  3 
     
  3 
 0 1 0
 2 –1   2
=  0 –1   –1  x  (–, –1)  2 , 
 0 0 0 
 3 3   0 0 3  3 3   3 
 
      (C) |2 – | [x] – 1| |  2
0 ||[x] – 1| – 2|  2 0  |[x] – 1|  4
 0 1   0 0 1  0 0 3
 
   – 3  [x]  5  x  [–3, 6)

 QT = Q +    
  = 0 Ans. (D) |sin–1 (3x – 4x3)|   –  sin–1 (3x – 4x3) 
2 2 2
1 2 0  – 1  3x – 4x3  1  –1x1

2 1 0
 115. (A) x  0 and g(x)  2  x  1
113. Given P =  Domain is x  (–, 0)  (0, 1)  (1, )
0 0 1 (B) x  2 and f(x)  0  x  1/2
Domain is x  (–, 1/2)  (1/2, 2)  (2, )
(C) f(f(x)) = x and x  2
 1 2 0   1 2 0  5 4 0  Range is (–, 2)  (2, )
     
2 1 0  2 1 0   4 5 0  2x  1
So P2 = PP =  =
0 0 1 0 0 1 0 0 1 (D) g(g(x)) = and x  0
x 1
Range is (–, 1)  (1, )
5 4 0   1 2 0 
    116. (A) Range of the function is (0, )
Now P 3
= 2
P P =  4 5 0  2 1 0  = (B) Function is decreasing
0 0 1 0 0 1 (C) Range of function is not R and function is one-one.
(D) The function is clearly many-one because the value of
13 14 0 function is zero at x =  and .
  And range cannot be R.
14 13 0
 0 0 1

RESONANCE Page - 35
MATHEMATICS

2n2  2n  1  2n2  2n  1  4 
 1 1 
117. (A) an =
cot   tan ( 2r  1)  tan ( 2r  1)
n 
 r 1 
4 cot (tan–19 – tan–1(1))
an = 2 2
2n  2n  1  2n  2n  1 cot (tan–1 4/5)

1 1  2n 2  2n  1  2n2  2n  1  3x  4
51x =
an = 4 
  3x  2

15x + 10 = 12x + 16
20 3x = 6
1

n1
=
4
 5  1 x=2

i2  i2 
 n
  n
(n 1) 
1 z e ...... z e
 13  5 ..................... 41  841 
n
118. We have z –1 = (z –1)      
+
4
761     

 i2   i2 
1  z – e n  ......  z  e n (n 1) 
= (29 – 1) = 7 1+ z +.....+ z = n–1
   ......(1)
4    
1 1 2  2
(C) – = Put z = 1 and take modulus n = 2n–1 sin sin ......sin (n–1)
sin x sin 2x sin 4 x n n
sin 2x  sin x 2 
sin x sin 2 x = 2 sin 2x cos 2x n

(P) If n = 21
2 cos 3 x / 2 sin x / 2 1
2 sin x / 2 cos x / 2 = cos 2 x  2 9 10 20
21 = 220 sin sin .......sin .sin ....sin
21 21 21 21 21
3x 2
cos cos2x = cosx/2   10  20
2 21   sin ........sin  .2
 21 21 
7x x x  10 21
cos + cos = 2 cos sin .......sin  10
2 2 2 21 21 2

7x x 3x (Q) If n = 22
cos = cos  sin 2x sin =0
2 2 2  2 10 
2

22  221  sin sin .........sin 
3x 3x  22 22 22 
sin 2x  0sin =0  = n 2
2 2   10  22 11
 sin .......sin   21  20
 22 22  2 2
2n
x= x
3  2 10  11
 sin sin ..... sin  10
2 4 8 10 22 22 22 2
x= , , , (R) Again put z = –1 is...............(1) and take modulus
3 3 3 3
 2 
 4 1  1  cos cos .......cos(n  1) 2 n 1
(D) cot
 tan 1
  n n n

 r 1 2r 2 
 n  21
 2 20 20
1  cos cos .......cos 2
 4  2   21 21 21
cot

tan 1 

 r 1  4r 2   
cos .......cos
10 1
 10
21 21 2
 4  (2r  1)  (2r  1)  

cot  tan 1 
 1  (2r  1)(2r  1)  
 r 1 

RESONANCE Page - 36
MATHEMATICS
1
  2 10   11 123. Putting x = 2, and – 1 successively
(S)  sin .sin .....sin   10 2
 22 22 22  2
 10 9   11
 cos cos ....cos   10
f (2)  f 
 1 
 22 22 22  2   3 . . . . . (1 )
 2 
 sin   cos(90  )
 1  3
f    f (  1)  .......( 2 )
p( x ) . g( x )  2  2
119. Let f(x) = a n d f (  1)  f ( 2 )  0 .......(3 )
h( x )
3
S o lv in g , w e g e t f ( 2 ) 
where p(x) = x  odd function 4
g(x) = log (x + x2  1 )  4f(2) = 3
g(x) + g(–x) = log (x + x 2  1 ) + log (–x + x2  1 )
= log (–x2
+ x2 + 1) 124. |z1| = 2 and (1 – i) z2 + (1 + i) z2  8 2
= log (1)
=0  g(x) is an odd function. C : x2 + y2 = 4 L:x+y= 4 2
x  e 1 x 1
h(x) = 
e   2 = e +
    2

x 1  x  1
h(x) + h(–x) = e + +   +
  2  e  2

 x   x 
= e +  e  + 1
   
= –1 + 1  [x] + [–x] = –1 if x  R – I AB = OB – r
=0 AB = 2.
h(x) is an odd function
 f(x) is an odd function
x 1 
 f(1947) + f(– 1947) + f(0) = 0.
125. Let f = 2
x  5x  9
f 
 11 ,1
 

1   1
120. Any point on the curve xy = 4 is
 2
 2t, 
 f2 + f + k < 0  f   11 ,1 f  11 < 0 & f(1) < 0
   
 t
126. Let |3sinx – 4cosx| = t
 t  [0, 5] x  R
 2 1 Given equation can be written as
slope of the tangent at  2t,  is –
 t  t2 t2 – (a2 + a + 5) t + a2(a + 3) + 2(a + 3) = 0
t2 – ((a2 + 2) + (a + 3)) t + (a + 3)(a2 + 2) > 0
 equation of tangent is x + t2y – 4t = 0  t=a+3 or t = a2 + 2
since it is tangent to x2 + y2 = 8 also  0a+35 or 0  a2 + 2  5
 –3  a  2 or –2  a2  3
4t
 = 2 2  –3  a  2 or – 3  a  3
1 t4  acceptable integer values of a’ are –3, –2, –1, 0, 1, 2
i.e. t2 = 1  6 is the answer
 equation of the tangent is x + y = ± 4
Since the intercepts are positive 127. Centre of circles c1, c2, c3 are in A.P.
 the tangent is x + y = 4 General term for abscissa of centres = 1 + (n – 1).3 = 3n – 2
 centre of c5 is (13, 0)
121. y = f(|x|) Radius of circles are in G.P.
 Rn = 1.2n – 1 = 2n – 1
 f ( x) , x  0  R3 = 4 and centre of c3 is (7, 0)
 y=  tangents of circle c3 intersect each other at (13, 0)
f (  x ) , x  0 equation of any line passing through (13, 0) is y – 0 = m(x – 13)
 mx – y – 13m = 0 now it will be required tangents if
f ( x ) , x  0
=  where t = –x and y = f(x) has no point 7m – 0 – 13m
 f (t ) , t  0
=4
of discontinuity when x  0 m2  1
y = f(|x|) has no discontinuous points

122. |a + b| = |a| + |b| 2


 ab  0 (x2 – 5x + 4) sinx  0  36m2 = 16m2 + 16  20m2 = 16  m = ±
 5
 x [0, 1]  [, 4]  {2}
sum of all integral values is 0 + 1 + 4 = 5

RESONANCE Page - 37
MATHEMATICS
 f (2) = – 4e–4 – 4e–4
2 2
Let m1 = , m2 = –  f (2) is negative hence at a = 2
5 5  f (a) is maximum
 10|m1m2| = 8

133. f'(x) has D > 0  36 – 4k2 > 0


 k  (–3, 3)
M1 M2 M3 M4 M5 M6 Ans. 3
L1    2
 1 1
L2    134. 2x – 2x + 1 = 2  x   +
2

128.  2  2
L3   
L4    1
 minimum value of 2x2 – 2x + 1 is and minimum value of
2
4 sin2x is 0
C1  3 C 2 4 2
Required probability = = – 2 x 1) sin2 x
6
C2 5  minimum value of e( 2 x is 1

Alter : Required probability = 1 – (no languages is common)


135. a = b
1 4 136. sin 3x = cos 4x
=1–1× =
5 5
   
129. Q1(x)  0  a > 0 and b2 – 4ac  0  cos4x = cos   3 x   cos4x = sinx ±   3 x 
Q2(x)  0  b > 0 and c2 – 4ab  0 2  2 
Q3(x)  0  c > 0 and a2 – 4bc  0 137. T r = cot –1 (2r2)
2
  a – 4(  ab)  0  
–1  1 
2 S=  cot –1( 2r 2 )   tan  2
  a  4(  ab)  a > 0, b > 0 and c > 0 r 1 r 1  2r 
  ab  0 
–1  ( 2r  1) – ( 2r – 1) 
a 2 =  tan  
 1  ( 2r  1)( 2r – 1) 
........(1) r 1
 ab  4

we know that (a – b)2 + (b – c)2 + (c – a)2 > 0 


–1
=  tan ( 2r  1) – tan –1( 2r – 1)
  a2 –  ab  0 r 1
2 = (tan–13 – tan–11) + (tan–15 – tan–13) + (tan–17 – tan–15) +.... +
a
 > 1 .........(2) tan–1
 ab
from (1) and (2), we can say that
    1
S=– + = Given S = = k  k =
4 2 4 4 4
 a2 [k] = 0
1<
 ab  4 138. f (x) = –2x2 . e–2x + 2x . e–2x
 possible of integers in the range are 2,3,4
 Number of integers in the range is 3. = 2(–x2 + x) e–2x
f (x) = 0 at x = 1 and f (x) > 0 in (0, 1) and f (x) < 0 in (1, )
 5 x.2  f(x) is maximum at x = 1
130. log10  x  = x  x = 1 only
 2  1 1 1
f(1) = 2 =   = e2
131. f(x) = 2x³ – 3 (k + 2)x² + 12kx – 7, – 4  k  6, k l e 
f'(x) = 6(x² – (k + 2)x + 2k) = 6(x – k) (x – 2) [–1] = 7
For f(x) to be invertible, k must be 2 only.
3
139. Slope of AB =
a 2
2–
2 Equation of AC
132. Let f(a) =  e – ( x – 2 ) dx
a–2 (2 –  )
y–0= (x – )
3
2 2
 f (a) = e –(a) – e –(a – 4) = 0
2 2
 e – a – e – ( a – 4 )  – a2 = – a2 – 8a + 16
 a = 2

2 2
f (a) = e – a (–2a) – e – ( a – 4 ) (–2(a – 4))

– a2 2  ( 2 –  ) 
= – 2a e + 2(a – 4) e –(a – 4)  C  0, 
 3 

RESONANCE Page - 38
MATHEMATICS
( 2 –  ) 1
sin x–1
Let M(h,k)  a = 2h, 2k = 144. Let  = x dx ......(1)
3 2
– x 1
0
 6k = 2h(2 – 2h)
1 1
3y = x(2 – 2x) sin –1 1– x sin–1 1– x
2x2 – 2x + 3y = 0 =  (1– x) 2
dx   dx
x2 – x  1

a = 2, b = 3 0
– (1– x)  1 0
a+b=5
1
140. x2 + (a – b) x + (1 – a – b) = 0 cos –1 x
 D>0 = x 2
dx ......(2)
0
– x 1
 (a – b)2 – 4 × 1 × (1 – a – b) > 0
a2 + b2 – 2ab – 4 + 4a + 4b > 0
 b2 + 2b (2 – a) + (a2 + 4a – 4) > 0  sin–1 x = cos–1 1– x 2
 4(2 – a)2 – 4 × 1 × (a2 + 4a – 4) < 0 (1) + (2) we get
4 + a2 – 4a – a2 – 4a + 4 < 0
 sin–1 1– x = cos–1 x
8a – 8 > 0
a>1 1
sin–1 x  cos –1 x
20 1 2=  dx
141. an = and d = x2 – x  1
n 20 0

1
20 20  1
20  2
dx
2 3 4 1  3 
2
0
 x – 2    2 
1 1/ 2 1/ 3 =
20 20 20
(20)3    
1 4/5 2/3
 = 4 5 6 =
20 20 20 47 1 7/8 7/9 1
7 8 9  1  –1  2x – 1 

  tan   
 = 4 3  3  0
applying R1  R1 – R2 and R2  R2 – R3
2
0 –3 /10 –1/ 3
(20)3 0 –3 / 40 –1/ 9      n2
 =
47 1 7/8  = 6  6 =  
7/9 2 3  2 3 3 108
 n = 108 
50
 =  [] = 2 Ans.
21 n
= 4 Ans.
142. Using the idea of the differentiation of determinant, we get 27
   
cos( x   ) cos( x  ) cos( x   ) 145. K r  r  a  b ..........(i)

f(x) = cos( x   ) cos( x  ) cos( x   ) + 


pre cross (i) with a both side
cos( –  ) cos(  –  ) cos( – )       
 K(a  r )  a  ( r  a )  a  b
   2      
sin( x   ) sin( x  ) sin( x   )  K ( a  r ) | a | r – ( a . r ) a  a  b
– sin( x   ) – sin( x  ) – sin( x   )    2      
 K(K r – b)  | a | r – (a . r ) a  a  b
cos( –  ) cos(  –  ) cos( – )
(using equation (i))
sin( x   ) sin( x  ) sin( x   )        
 r (K 2  | a |2 )  a  b  Kb  (a . r ) a ..........(ii)
+
cos( x   ) cos( x  ) cos( x   ) 
Dot (i) with a both side
0 0 0    
=0+0+0 K(a . r )  a . b
f(x) = 0 for all x
f(x) = a constant  
  a.b
But f(9) =   f(x) =  for all x.  a. r  ..........(iii)
K
9
by (ii) & (iii)
 f(k)
k 1  1
 
  a .b    
=9 r    Kb  a  a  b
 K 2  | a |2  K 

143. Put log10x = t
[t2 + t] = 2t + 1  mn=3
2t + 1  t2 + t < 2t + 2
and 2t  
t

RESONANCE Page - 39
MATHEMATICS
146. For defining f(x)
(16 – x), (20 – 3x)  N ; (2x – 1), (4x – 5)  W  f(x) =  F(x)  xF'(x) dx
(16 – x)  (2x – 1) ; (20 – 3x)  (4x – 5)
Hence x = 2, 3 x = 2, 3  f(x) = x F(x) + c
when x = 2, f(x) = 14C3 + 14C3 = 728
when x = 3, f(x) = 13C5 + 11C4 = 1617 2      
Hence sum 728 + 1617 = 2345 f =    f = F  + c
4  2 2 2 2
147. z =  (z – z2 + 2z3)(2 – z + z2) = ( – 2 + 2) (2 –  + 2 )
1
2  
= |2 –  + 2 |2 = | 2 – i 3 |2 = 7 = 2 +cc=0
4 2  
148. On expanding the determinant, we get  f(x) = xF(x)
7sin3 + 14cos2 – 14 = 0  f() = x F() = ()0
 sin3 – 2(1 – cos2) = 0  3sin – 4sin3 – 4sin2 = 0  |cos(f())| = |cos | = 1

1 152. [x + a] = [x] + a ; if a  I
 sin (2sin – 1)(2sin + 3) = 0  sin = 0 or sin =
2
 2[ x ] 
 5 Minimum if 'a' for sin   is 3
  = 0, , 2, ,  5 solutions  3 
6 6
 3[ x ] 
149.  1 lies between the roots and for cos   is 4
hence f (1) < 0  2 
 3 – 3 sin  – 2 cos2  < 0  2 sin2  – 3 sin  + 1 < 0 T = 12

1  5 16 4
2
< sin  < 1 2n + << + 2n, n  I  2 8 x 16   16
= 3  x   +  –

2 6 6 153. f (x) = 3  x    +–
 3 9  3  3 3
  5    
In [0, 2],   ,  –   2 4 4 2
 6 6  2 now – 2 < x < – –2+
3
<x+
3
<
3 3
 = 1, 2
Required sum 1 + 2 = 3 2
2  4 2  4 4
       < x   <  x   <
150. Volume = |[3( b  c ) 2( a  b ) 4( c  a )]| = 18 –
3  3 3  3 9

   2 18 4 16
[a b c ] = 24  f (x) < 3 . +–
9 3
3   4
  
[a  greatest value of  is 4
b c] = ±
2 ...(1)

  
[a b c ]  3x   
154. f(x) = n  2 sin x  tan x –  1 , x   , 
   6 3
1  sin  cos  sin 2
 2   2   4  3x
sin    cos    sin 2    
3  3  3  Let g(x) = 2sinx + tanx – + 1, x   , 
=     6 3
 2   2   4 
sin    cos    sin 2  
 3   3   3  3  
g(x) = 2cosx + sec2x – > 0,  x  6, 3
(R1  R1 + R2 + R3)
  
 g(x) is strictly increasing 
3
=  cos3       
2 range of g(x) is g   , g   
cos3 = 1   6   3 
3 = 
1 values of        
 g(x) g   , g   
sin x
  6   3 
151.  f(x) = x (1 + xcosx . nx + sin x)dx
 1 3 
i.e.   , 2 3
 sin x  sin x    3 2

=  x  x.x sin x  cos x ln x  
x   dx

 
Let F(x) = xsinx  3 1  
range of f(x) is  n    , n(2 3 )
 1    2 3 
 F’(x) =  sin x. x  cos.ln x  xsinx
 

RESONANCE Page - 40
MATHEMATICS
 3 1
1    1 1.x 
 f(x)  n    , n(2 3 ) =
2  x sin –1 x
  – dx 
  2 3  

0
0 1– x 2 

3 1   1 1
 a+b= n    , n(2 3 ) – 
2 3 2  – (1– x 2 ) 2 x dx 
 =
2
0
 
 
= n( 3 3  2) 
[a + b] = 1 1 1

=+  (1– x 2 ) 2 (–2x)dx
155. +k, –x2 + kx + 2 is –ve for infinitely many integers.
0
156. The triangle has circumcentre at origin and its orthocentre lying
on the circumcircle. 1
=  2  1– x 2 
157. R(a + b) = c 0
ab
=  + 2(0 – 1) =  – 2
R(a + b) = 2R sinC ab  a = 1, b = – 2  a – b = 3

ab  1 1 1 
160. z 1 z 2 + z 2 z 3 + z 3z 1 = z 1z 2z 3    = 1
2 ab
= sinC
 z1 z 2 z3 

ab  | z1 |2 | z 2 |2 | z 3 |2 
 
z3  = z1 + z2 + z3
we know  1 and sinC  1  
2 ab  z1 z2

 a+b=2 ab and sinC = 1 = ( z1  z2  z3 ) = 1


 The cubic equation with roots z1, z2 and z3 will be
A z3 – (z1)z2 + (z1z2)z – z1z2z3 = 0
 z3 – (1)z2 + (1) z – 1 = 0
)

2a  (z – 1)(z2 + 1) = 0
  z = 1, i, – i
c b
 Im(z1) < Im(z2) < Im(z3)
)  z1 = – i , z2 = 1, z3 = i
B a C  |z1 + z 22 + z33 | = |–i + 1 –i| = |1 – 2i| = 5 [ 5 ]=2
 a = b and c = 90°
x
1 161. Let f = 2
a.a x  5x  9
 2 a
r= = =
s aaa 2 2 2 1 
f 
 11 ,1
2  

158. f ''(x) –f '(x) =ex 1 


Put f '(x) =  f2 + f + k < 0  f   11 ,1
 
d
 (–1)  e x  e – x  e x . e – x dx
z 1
dx 
 t   < 0 & f(1) < 0
 e – x  x  C 1, f '( 0 )  1  C 1  1
 11
f '(x) = xex + ex 
162.  r = =1=s
f(x) = xex + C2  f(0)  0  C 2  0 s
(PB)(PC) = (s – b)(s – c)
 f ( x)  xe x  f (2)  2e 2
2
s(s – a)(s – b)(s – c)
F (f(2)) IJ  4
inG
=
s(s – a)
H 4 K
2  
= = .
s(s – a) s s – a
159. A

Required area = 2 sin–1 xdx



0 P
L
B * C
s–b s–c

RESONANCE Page - 41
MATHEMATICS

x3  y3  z3 xyz  x 2  y 2  z2 
= r.  r = 1 and s =    .....(1)
–a   
3  3   3 

(PB)(PC) = ........ (1)
–a
x 2  y 2  z2 xyz xyz
    .......(2)
1 3  3  3 
= (BC) (height)
2
3
1 3a x 3  y 3  z3  x  y  z 
    ...........(3)
= (s – b + s –c) (3) = 3  3 
2 2
3 3 3
3a / 2 (x  y  z)2 x  y  z
Given = ...........(4)
 from equation (1) (PB)(PC) = 3a 2 3
–a =3
2
27
Alter (3), (4)  x + y + z 
Let circle be x2+ y2 = 1 2
Pair of tangents from A(, –2) is If x + y + z = 13, x,y,z will not satisfy (4)
(x2 + y2 –1) (2 + 3) = (x – 2y – 1)2 If x + y + z = 12, then/ x = 3,y = 4, z = 5 will not satisfy (4)
Hence maximum value is 12.
x=0
164. given that f 2010   2008, 2012 
B(p,1) P(0, 1) C(q,1)
y=1
r=1

y=0
f 2010 
O   200 .8, 201.2
10
3

 f 2010 
  10   200,201
 
A(,–2)

x2(2 + 3) = (x – 3)2 [put y = 1 for points B and C]  x   f (x) 


Nowx – f(x) = 20   – 10  
 20   10 
 3x2 + 6x – 9 = 0
q Put x = 2010
 pq = – 3  (PB)(PC) = 3
 2010 
163. Let f(t) = t3, u = f(t) 2010 – f(2010) = 20 ×  20  – 10 × (200 or 201)
A(x, x3), B(y, y3), C(z, z3)  
Centroid G lies inside ABC = 20 × 100 – 10 × (200 or 201)
GL  ML = 2000 – [2000 or 2010]
= 0 or – 10
x3  y3  z3 xyz
 f   not possbile
3  3 
165. (x – r)2 + y2 = r2

x2 y2
 =1
a2 b2
P(acos, bsin)  (acos – r)2 + (bsin)2 = r2 ..........(1)

x3  y3  z3 ( x  y  z )3
 .......(1)
3 27
3 3 3
( x  y  z )2 x  y  z
Given = ......(2)
2 3

( x  y  z ) 2 ( x  y  z )3 27 Normal passes through (r, 0)


  (x + y + z) 
2 27 2 a.r b.0 r
If x + y + z = 13, x,y,z will not satisfy (2) – = a2 – b2  cos =
If x + y + z = 12, x = 3,y = 4, z = 5 will not satisfy (2)
cos  sin  ae 2
Hence maximum value is 12.
Alternate : r2 r
put in (1) b2 = 2  b=
e e
a  b  c  a  b  c        
   
3  3  3   b2 r 2
Area = ab = = differentiable w.r.t. e
1– e2 e2 1 – e2

RESONANCE Page - 42
MATHEMATICS
2  3  3i  2   1  3i 
e = arg   = arg  
3  3  3i  2i   3i 
166. y = x3 ....................................(1)
let P(h,k)  k=h3  P(h,h ) 3

dy (3, 3)
y  x3   3x2 (0, 2)
dx
 Slope of tangent at P  3h2
equation of tangent at P(h,k) (2, 0)

y  h 3  3h 2 (x  h)

 y  3h2 x  2h3 ........................(2)


Solving 1 & 2

x3  3h2 x  2h3  0 1
= tan13 tan1
3
(x  h)2 (x  2h)  0
 x   2h& y   8h3  point
 3  1/ 3 
= tan1  
 2 
P1 ( 2h, 8h3 )
= tan1  8
Now tangent at P1(( 2h),( 2h)3 ) will meet the curve at 6
 
P2 ( 2( 2h), 8( 2h)3 )
4 
i.e P2 (4h,64h3 ) = tan1

3 11
similarly P3 (8h,–512h3 ) Now : From the figure it is clear that if rays don’t
Area intersect the circle.

2h 8h3 1 1 1 1  1 1 1 
1 then      tan 3 ,   tan 3
of  P1 P2 P3  4h 64h3 1  8h4 2 8 1  
2
8h –512h3 1 4 64 1
1
 |a+b|= 2  tan1 3  tan1
h h3 1 1 1 1 3
1 3 h4
Area of  PP1 P2  2h 8h 1  2 8 1 
2 2 = 2 
4h 64h3 1 4 64 1 2
areaof  P1 P2P3 3
  16 =
area of  PP1 P2 2
 10 | a + b | = 15 
167. Replace n by n – 1
 [ 10 | a + b | ] = [ 15 ] = 47
1 1 1
+ +......+ = f(n)
f (0)  f (1) f (1)  f (2) f (n – 1)  f (n)
 3x   
put in (b) 169. f(x) = n  2 sin x  tan x   1 x   , 
   6 3
1
f(n) + = f(n +1) 3x
f (n)  f (n  1) consider g(x) = 2 sin x + tan x – +1
 1 = f2(n + 1) – f2(n) 
 1 = f2(1) – f2(0)
 1 = f2(2) – f2(1) 3  
g(x) = 2 cos x + sec 2x – > 0 for all x   , 
  6 3
1 = f2(100) – f2(99)  g(x) is monotonically increasing
Adding
100 = f2(100) – f2(0)  f(0) = 0    3 
g   = 2 sin + tan – . +1
2 2
Also n = f (n) – f (0)  f(n) = n 6 6 6  6
f(9801) = 99
1 1
168. First note that (3, 3) is in the interior of the arc, as =1+ – +1
3 2

RESONANCE Page - 43
MATHEMATICS
a1, a2 , a4  {2,4,8}, a5 , a6 , a7  {3,5,6}
 3 1  or
 f   = n   
 a1, a2 , a4  {3,5,6}, a5 , a6 , a7  {2,4,8}
6 2 3 Required number of ways = (3! × 3!) + (3! × 3!) = 72

 174. f(x) = sin (n x) – cos (n x), x > 0


f   = n
3
 3 
3  1  1 = n 2 3
 
= 2 sin  nx – 
 4
3 1 
 a + b = n    + n 2 3
2 
 3 2  
f(x) = cos nx – 
x  4
 3
  1  2 3 
 
= n  2
 3 

= n 3 
3  2 = n 7.2 2  
sin   n x 
=
x  4 
 1<a+b<2  [a + b] = 1

170. g(x) = x[f(2x2 – 1) – f(1 – x2)]  


Case–I x > 0
f(x)  0  sin  4  n x   0
For g(x) to be strictly increasing f (2x2 – 1) – f (1 – x2) > 0
 
 f(2x2 – 1) > f(1 – x2)  2x2 – 1 > 1 – x2  3x2 > 2   3
2  0 +n x    –  nx 
2 4 4 4
 x2 >  x>
3 3  – =   – 5 cos (– ) = 5
( x > 0)
Case–II x < 0
For g(x) to be strictly increasing f (2x2 – 1) – f (1 – x2) < 0
sin 3 x cos 5 x – sin 3 x cos 4 x
175.  dx
 f (2x2 – 1) < f (1 – x2)  2x2 – 1 < 1 – x2  3x2 < 2 sin 3 x cos 3 x  sin 3 x
2

2 2
1 (sin 8 x – sin 2x ) – (sin 7 x – sin x )
2 2
=  dx
 x2 < – <x< – <x<0 2 sin 6 x  sin 3 x
3 3 3 3

 2   2  9x 7x 9x 5x
Thus g(x) strictly increases on the interval   3 , 0    3 ,   2 sin cos – 2 sin cos
  
    1 2 2 2 2 dx
171. 3 + 4 cos x = t  – 4 sin x dx = dt
= 2  9x
2 sin cos
3x
2 2
1 t3
I  dt
8 t3 x
2 sin 3 x sin
1 2 dx
– 3x x
1 1 3 
  dt

8 t2 t3
= 2
cos
3x =–  2 sin 2 sin
2
dx
2
11 3  sin 2x
e
  
8  t 2t 2 
=  (cos 2x – cos x) dx = 2
– sin x + C

a=2
2t  3 8 cos x  3
  e 176. f(x) = 12cos3 x(– sin x) + 30 cos2 x(– sin x) + 12 cos x(– sin x)
16 t 2 16(3  4 cos x )2 = – 3 sin 2x(2cos2 x + 5 cos x + 2)
 a = 3, b = 8, c = 16, d = 2 = – 3 sin 2x (2cos x + 1) (cos x + 2)
172. f(x) = x3 – x2 + 100x + 1001 
When, f(x) = 0  sin 2x = 0x = 0, ,
f (x) = 3x2
– 2x + 100 > 0 xR 2
 f(x) is increasing (strictly)
2
or, 2 cos x + 1 = 0  x=
 1   1  3
 f  > f 
 1999   2000  or, cos x + 2 = 0 (not possible)
Sign scheme for f(x) in [0, ] is as below.
f(x + 1) > f(x – 1)

    2 
f(x) decreases on  0,    ,  
 2  3 
173.
  2 
and increases on  , 
(a1 + a2 + a3 + a4) = 21, 2 3 
a5 + a3 + a6 + a7 = 21,
a1 + a2 + a3 + a4 + a5 + a6 + a7 = 35
 a3 = 7, a1 + a2 + a4 = 14, a5 + a6 + a7 = 14

RESONANCE Page - 44
MATHEMATICS
177. Number of selections of 7 digits out of the digit 1,2,3,.....,9 = 9c7 179. (n3 – 1)3 = (n + 1)3(n – 1)3
Number of digits out of these 7 selected digits excluding the (n + 1)3 – (n – 1)3 = 6n2 + 2
greatest digit = 6
These 6 digits can be divided in two group each having 3 3 3
3n2  1 2
1 6n  2 1  (n  1) – (n – 1) 
2 3 = = 3 3

6! 6 1 (n – 1) 2 (n2 – 1)3 2  (n  1) (n – 1) 
digits   C3 x
3 ! 3 ! 2! 2!

But the 3 digits on one side can go on the other side  1 1 


1  
 Required number of ways = =  (n – 1)3 – (n  1)3 
2  
9 6 2 9 6 9 6
C7 . C 3 .  C 7 . C3  C2 . C3 = 720 S=
2!
1  1 1   1 1   1 1   1 1  
178. b  40  12  3 – 3    3 – 3    3 – 3    3 – 3   ...
2  1 3   2 4   3 5   4 6  
c  3   10

cos1 cos x  x  a  b  c 1 1  1  9 9
=  =  16S = 9 16 =
2  8  16 S
 cos1 cos x  x  
1/n
A  (2n  1)(2n  2)(2n  3).......(2n  n) 
180. = lim  
B n 
 (n  1)(n  2)(n  3).........(n  n) 

 r 
1 n  2 n 
lim  ln  
ln (A/B) = n  n r 1  1  r / n 
 
Integral value of x satisfying given equations = 2,3,4

1
 (2  x)  27
0 ln   dx =
 (1  x)  16
 A = 27 & B = 16

RESONANCE Page - 45
We Are Here For You...
Study Centres
KOTA Alambagh Campus Boisar Campus RANCHI
Corporate/Head Office II Floor Basant Tower, Opp. One Up Motors, Near Dr. Homi Bhabha Vidyalaya, 1st Flr, 501, Le-Desire Building, Opp Hari Om Towers,
CG Tower, A-46 & 52, IPIA, Near City Mall, Jhalawar Avadh Hospital, Alambagh, Lucknow Anushree Colony, TAPS, Tarapur, Boisar - 401504 Circular Road, Ranchi - 834001
Road, Kota (Raj.)-324005 Tel.: 0522-3206974, 3192223, 6060662 Tel.: 09321313417 Tel.: 0651-6060660, 7677923007
Tel.: 0744-3012222, 3192222, 6635555 e-mail: lko@resonance.ac.in e-mail: tarapur@resonance.ac.in e-mail: ranchi@resonance.ac.in
Fax: 022-39167222, 744-2427144
Website: www.resonance.ac.in KOLKATA Lodhivali Campus ALLAHABAD
e-mail: contact@resonance.ac.in 1-A.J.C. Bose Road, 2nd Floor, Near Exide More & Resonance, A/4, Reliance Township, 4th Floor, Vinayak Tower, Above Axis Bank,
Lord Sinha Road Crossing Lodhivali Township, Lodhivali, Navi Mumbai - 400705 M.G.Road, Civil Lines, Allahabad-211001
Commerce Division Campus: (Entrance through Lord Sinha Road) Tel.: 07709833704 Tel.: 0532-6060660, 9984255086
607/608-A, Talwandi, Kota (Raj.) - 324005 Kolkata (West Bengal) - 700 020 e-mail: lodhivali@resonance.ac.in e-mail: allahabad@resonance.ac.in
Tel.: 0744-3192229, 9529980666 Tel.: 033-31922222, 60606600
e-mail: commerce@resonance.ac.in email: kolkata@resonance.ac.in UDAIPUR GWALIOR
Resonance, First Floor, B-Block, Anand Plaza, 1st & 2nd Flr, Bansi Plaza, Near S.P. Office,
PCCP/PSPD/MEx Campus NAGPUR University Road, Udaipur (Raj) -1 City Centre, Gwalior - 474012
J-2, Jawahar Nagar Main Road, Kota (Raj.) Main Campus Tel.: 0294-6060660, 5107858 Tel.: 0751-6060660, 9669688934
Tel.: 0744-2434727, 8824078330 Pragati Building, 2nd Floor, Sanskrutik Sankul Building, e-mail: udaipur@resonance.ac.in e-mail: gwalior@resonance.ac.in
e-mail: pccp@resonance.ac.in Jhansi Rani Square, Sita Buldi, Nagpur - 440012
Tel: 0712 - 3017222, 3192222, 6060660. University Road Campus
JABALPUR
Station Campus (PCCP/Commerce only) Mob.: 9372137857, 9373507997 / 8 A-One School, University Road, Udaipur-1
1st Floor, Nayan Tower, Near Madan Mahal Police
1st Floor, C/o 3-4, Vidya Vihar, Manoj Marg, Station e-mail: nagpur@resonance.ac.in Tel.: 0294-3192222, 2427858,
Station, Shivaji Chowk, Napier Town - 2
Road, Kota (Raj.)-324002
Tel.: 0761-6060660, 8120005717
Tel.: 0744-3192223, 9829329593, 9828545223 Extension Campus BHUBANESWAR e-mail: jabalpur@resonance.ac.in
Vinamra Building, Above PNB Bank, Beralia House. Plot No-21/5, Near Regional Science Centre,
JAIPUR Plot No. 91, Chaprunagar Square, Central Avenue Acharya Vihar, Bhubneshwar-751013
AURANGABAD
Central Campus Road, Nagpur - 440 008 Tel.: 0674-3192222, 3274919, 6060660/1
2nd Floor, Alaknanda Building, Near Baba Petrol
B L Tower Ist, Near Jaipur Hospital, Mahaveer Nagar Tel: 0712-6462622, 3922333, 6060661 e-mail: bbsr@resonance.ac.in
Pump, Adalat Road Aurangabad - 431001
1st, Tonk Road,
Tel.: 9910066021
Jaipur (Raj.) - 302 015 MUMBAI AHMEDABAD e-mail: aurangabad@resonance.ac.in
Tel.: 0141- 6060661, 6060664, 3192222 Andheri Campus (Regional Office) 2nd Floor, C-Block, Galaxy Bazaar,
e-mail: jaipurc@resonance.ac.in Spenta Mansion, 2nd Floor, Above Moti Mahal Hotel, Opp. Sunrise Park, Near Himalaya Mall,
Opp Station, Andheri (W), Mumbai - 400 058 Off Drive-In Road Vastrapur, Ahmedabad RAIPUR
West Campus Tel.: 022-3192 2222, 60606600 Tel.: 079-31922222/3, 60606600/1/2 1st & 2nd Floor, Vandana Auto Building, Opp Raj
B-99, Near National Handloom, Vashali Nagar, Jaipur e-mail: andheri@resonance.ac.in e-mail: abad@resonance.ac.in Kumar College, Raipur Pin code- 492001
(Raj.) BRANCHES: SHAHIBAUG I MANINAGAR I Tel.: 0771-6060660, 7771007840
Tel.: 0141-3103666, 4042188 Borivali Campus GANDHINAGAR e-mail: raipur@resonance.ac.in
e-mail: jaipurw@resonance.ac.in 2nd Floor, Raghuvir Tower, Junction of SVP Road &
Ganjawala Lane, Chamunda Circle, PATNA NASHIK
East Campus (PCCP only) Borivali (W), Mumbai Kankarbagh Campus Mangal Plaza, Second Floor, Near Kalika Mandir,
Plot No: 188, Gali No. -1, Panchwati Circle, Raja Tel.: 022-31922226, 6060 6601 NC/8, Gayatri Mandir Road, Kankar Bagh, Old Agra Road, Nashik-422002
Park, Jaipur (Raj.) e-mail: kandivali@resonance.ac.in Patna (Bihar) - 800020 Mob.: 09730877804
Tel.: 0141-3101166 Tel.: 0612-3192222, 2350070 e-mail: nashik@resonance.ac.in
e-mail: jaipure@resonance.ac.in Churchgate Campus e-mail: patna@resonance.ac.in
Jai Hind College, Admissions Office "A" Road, CHANDRAPUR
North Campus (PCCP only) Churchgate Mumbai-400020 (Maharashtra) Boring Road Campus Resonance Study Centre, Zanzari Complex, Z.P. Road,
ARGHAM Tower, R-6 (B-1), Tel.: 022-31922223, 60606602 3rd Floor A G Palace, Besides St. John's High Near Hotel Rasraj, Chandrapur-442401
Sector 1 Near MGPS School, e-mail: jaihind@resonance.ac.in School, Near Panchmukhi Hanuman Mandir,
Vidhyadhar Nagar, Jaipur (Rajasthan) East Boring Canal Road, Patna SURAT
Tel.: 0141-3192224, 6060662, 6060663 Dadar Campus Tel.: 0612-3192223, 2520109 21st Century Business Centre, Near World Trade
e-mail: jaipurn@resonance.ac.in C 8 (A) Kasturchand Mill Compound, Ground Floor, e-mail: patna@resonance.ac.in Centre, Udhna Darwaja, Ring Road, Surat
Near Manish Market, Senapati Bapat Marg, Near Tel.: 7874438681, 09725552181
BHOPAL Dadar west Railway station,Dadar (W), Mumbai JODHPUR e-mail: surat@resonance.ac.in
224, Smriti Complex, Zone-1, M.P. Nagar, Bhopal Tel.: 022-31922224 / 9769210458 Aanchal Complex (Basement), Near Manidhari
(M.P.) - 462011 e-mail: dadar@resonance.ac.in Hospital, Residency Road, Sardarpura, Jodhpur RAJKOT
Tel.: 0755-3206353, 3192222 Tel.: 0291-3192222, 6060660 2nd Floor, Gayatri Commercial Complex, Opp. Parimal
Fax: 0755 - 4044453 Govandi Campus e-mail: jodhpur@resonance.ac.in School, Kalavaad Road, Rajkot-360005
e-mail: bhopal@resonance.ac.in 1st Floor, Runwal Complex, Opp ICICI Bank, Tel.: 09314805678, 09974802362
Govandi Station Road, Govandi (E), Mumbai e-mail: rajkot@resonance.ac.in
AJMER
NEW DELHI Tel.: 022-31922225, 9930269183
Plot No. 1408, Aryan College Building, Near Ram
South Campus e-mail: govandi@resonance.ac.in
Bhawan, Shastri Nagar, Ajmer-305001 VADODARA
41 A, Ground Floor, Kalu Sarai, Sarvapriya Vihar, Tel.: 0145-3192222, 2621614, 6060660
Nerul Campus Royal Hub, Next to Seven Seas Mall, Fatehgunj,
New Delhi - 110016 e-mail: ajmer@resonance.ac.in
C-11, Amrita Sadan, Plot no. 13/14, Sector - 22, Vadodara - 390002
Tel.: 011-31922222, 60606602
Oppo. Nerul Rly. Station, Nerul (W), Navi Mumbai-6 Tel.: 0265-6060660, 7567810117
e-mail: delhi@resonance.ac.in
Tel.: 022-31922227, 60606605 INDORE e-mail: vadodara@resonance.ac.in
e-mail: nerul@resonance.ac.in 1st Floor, Gravity Tower, Opp. Agrawal Stores,
North West Campus
10, Ground Floor, Central Market, Punjabi Bagh
Indore - 452003 (MP) VISAKHAPATNAM (ICCP)
Panvel Campus Tel: 0731-3192222, 4274200 Vikas Vidyaniketan, Door No.: #1-1-1/1, Sheela
(West), New Delhi - 110026
Deep Dhara, Plot No.08, Road No.18, Sector-1, e-mail: indore@resonance.ac.in Nagar, NH-5, Visakhapatnam, (AP) - 530012
Tel.: 011-31922224, 60606601
e-mail: delhinw@resonance.ac.in Opp Geetanjali Building, New Panvel - 410 206 Tel: 0891-2757575
Tel.: 022-31922232, 60606606 SIKAR e-mail: vizag@resonance.ac.in
East Campus e-mail: panvel@resonance.ac.in Vishwakarma Apartment, Piprali Road, Sikar-332001
14, Pratap Nagar, Mayur Vihar Phase-1, Tel: 01572-319222, 606066 & 258222 BHAVNAGAR (ICCP)
New Delhi - 110096 Thane Campus e-mail: sikar@resonance.ac.in Sardar Patel Educational Institute,
Tel.: 011-31922223 / 5, 60606603 Paranjpe Udyog Bhavan, 2nd floor, Above Khandelwal Kalvibid, Bhavnagar 364002 (Gujarat)
e-mail: delhie@resonance.ac.in Sweets, Opp. station Thane (W) - 400607 AGRA Mob.: 8000816767 / 8000716868
Tel.: 022-31922228, 32191498, 6060 6607 1, Kailash Vihar, Behind St. Conrad's college,
LUCKNOW e-mail: thane@resonance.ac.in Khandari, By-Pass Road, Agra-282005 DEHRADUN (ICCP)
Hazaratganj Campus Tel: 0562-3192222 SelaQui International School, Chakrata Road,
13/1, Moti Chamber, Rana Pratap Marg, YMCA Kharghar Campus e-mail: agra@resonance.ac.in Dehradun - UK, INDIA
Complex, Ground Floor, In Front of Gauri Apartment, 5/6/7/8, Swami Samarth Bldg., Sector-12, Tel: 91-135-3051000 | Mob.: 9873743433
R.F. Bahaduri Marg, Hazratganj, Lucknow (U.P.) - 1 Opp. Kendriya Vihar Colony, Behind Syndicate Bank, NANDED
Tel.: 0522-3192222, 3205854, 6060660/1 Behind Mercantile Bank, Kharghar Beside Akola Urban Cooperative Bank, V.I.P Road,
Mob.: 9369225676 Tel.: 022-32191499, 60606608 Paras Square, Nanded.- 431602
e-mail: lko@resonance.ac.in e-mail: kharghar@resonance.ac.in Tel: 02462-250220, 606066
e-mail: nanded@resonance.ac.in

CORPORATE OFFICE: CG Tower, A-46 & 52, IPIA , Near City Mall, Jhalawar Road, Kota (Rajasthan) - 324005
Reg. Office: J-2, Jawahar Nagar Main Road, Kota (Raj.) - 324005 | Tel. No.: 0744-3192222, 3012222, 3022222 | Fax : 022-39167222, 0744-2427144
To know more: sms RESO at 56677 | Toll Free : 1800 200 2244 | Website: www.resonance.ac.in | e-mail: contact@resonance.ac.in

You might also like